Você está na página 1de 112

Problemas Sistematizao e

Representao
Licio Hernanes Bezerra
Carmem S. Comitre Gimenez
Nereu Estanislau Burin

2 Edio
Florianpolis, 2009

Governo Federal
Presidente da Repblica: Luiz Incio Lula da Silva
Ministro de Educao: Fernando Haddad
Secretrio de Ensino a Distncia: Carlos Eduardo Bielschowky
Coordenador Nacional da Universidade Aberta do Brasil: Celso Costa

Universidade Federal de Santa Catarina


Reitor: Alvaro Toubes Prata
Vice-Reitor: Carlos Alberto Justo da Silva
Secretrio de Educao a Distncia: Ccero Barbosa
Pr-Reitora de Ensino de Graduao: Yara Maria Rauh Mller
Pr-Reitora de Pesquisa e Extenso: Dbora Peres Menezes
Pr-Reitor de Ps-Graduao: Jos Roberto OShea
Pr-Reitor de Desenvolvimento Humano e Social: Luiz Henrique Vieira Silva
Pr-Reitor de Infra-Estrutura: Joo Batista Furtuoso
Pr-Reitor de Assuntos Estudantis: Cludio Jos Amante
Centro de Cincias da Educao: Carlos Alberto Marques
Centro de Cincias Fsicas e Matemticas: Mricles Thadeu Moretti
Centro de Filosofia e Cincias Humanas: Maria Juracy Filgueiras Toneli

Cursos de Licenciaturas na Modalidade Distncia


Coordenao Acadmica Matemtica: Neri Terezinha Both Carvalho
Coordenao de Ambientes Virtuais: Nereu Estanislau Burin
Coordenao de Infra-Estrutura e Plos: Vladimir Arthur Fey

Comisso Editorial
Antnio Carlos Gardel Leito
Albertina Zatelli
Elisa Zunko Toma
Igor Mozolevski
Luiz Augusto Saeger
Roberto Corra da Silva
Ruy Coimbra Charo

Laboratrio de Novas Tecnologias - LANTEC/CED


Coordenao Pedaggica
Coordenao Geral: Andrea Lapa
Coordenao Pedaggica: Roseli Zen Cerny
Ncleo de Formao: Nilza Godoy Gomes
Ncleo de Pesquisa e Avaliao: Claudia Regina Flores

Ncleo de Criao e Desenvolvimento de Materiais


Design Grfico
Coordenao: Laura Martins Rodrigues, Thiago Rocha Oliveira
Projeto Grfico Original: Diogo Henrique Ropelato, Marta Cristina Goulart
Braga, Natal Anacleto Chicca Junior

Redesenho do Projeto Grfico: Laura Martins Rodrigues,


Thiago Rocha Oliveira

Diagramao: Thiago Rocha Oliveira, Laura Martins Rodrigues


Ilustraes: Maximilian Vartuli, ngelo Bortolini Silveira
Capa: Natlia de Gouva Silva

Design Instrucional
Coordenao: Juliana Machado
Design Instrucional: Alessandra Zago Dahmer
Reviso de Design Instrucional: Mrcia Maria Bernal
Reviso Gramatical: Maria Tereza de Queiroz Piacentini
Copyright 2009, Universidade Federal de Santa Catarina/CFM/CED/UFSC
Nenhuma parte deste material poder ser reproduzida, transmitida e gravada, por qualquer
meio eletrnico, por fotocpia e outros, sem a prvia autorizao, por escrito, da Coordenao
Acadmica do Curso de Licenciatura em Matemtica na Modalidade Distncia.

Ficha Catalogrfica
B574p Bezerra, Licio Hernanes

Problemas: sistematizao e representao / Licio Hernanes

Bezerra, Carmem Suzane Comitre Gimenez, Nereu Estanislau

Burin 2. ed. Florianpolis: UFSC/EAD/CED/CFM, 2009.

112 p.

ISBN 978-85-99379-67-7


1.Matemtica. I. Gimenez, Carmem S. Comitre. II. Burin, Nereu

Estanislau. III Ttulo.

CDU 51
Elaborada pela Bibliotecria Eleonora M. F. Vieira CRB 14/786

Sumrio
Apresentao.............................................................................. 7
Introduo................................................................................... 9
1. Exerccios de Sistemtica e Representao..................... 13
1.1 Exerccios de Sistemtica............................................................ 15
1.1.1 Exerccios Propostos............................................................ 19
1.1.2 Soluo dos Exerccios Propostos...................................... 23
1.2 Exerccios de Representao...................................................... 26
1.2.1 Exerccios Propostos........................................................... 27
1.2.2 Soluo dos Exerccios Propostos..................................... 29
Referncias......................................................................................... 32

2. Decifrando Enigmas uma homenagem


a Malba Tahan.................................................................... 33
2.1 Introduo.................................................................................... 35
2.2 Enigmas, quebra-cabeas, dobletes, tripletes.......................... 35
Soluo dos problemas pares.................................................... 40
Referncias......................................................................................... 43

3. Problemas Olmpicos.......................................................... 45
3.1 Introduo.................................................................................... 47
3.2 XXVI OBM Nvel 1 1 fase.................................................... 48
3.3 XXVI OBM Nvel 2 1a fase.................................................... 55
3.4 XXVI OBM Nvel 3 1 fase.................................................... 60
3.5 Problemas olmpicos - miscelnea............................................ 66
Soluo dos problemas 1 a 46.................................................... 73
3.6 Problemas Propostos ................................................................. 82
Referncias......................................................................................... 85

4. Raciocnio Lgico................................................................ 87
4.1 Introduo.................................................................................... 89
4.2 Mtodo axiomtico..................................................................... 91
4.3 Clculo Proposicional................................................................. 94
4.4 Frmulas abertas......................................................................... 98
4.5 Quantificao............................................................................... 99
4.6 Exerccios Propostos................................................................. 103
Referncias........................................................................................110

Sugestes de Leitura............................................................. 111


Bibliografia Comentada....................................................... 112

Apresentao
Caro aluno,
A disciplina Problemas Sistematizao e Representao trata
basicamente de resoluo de problemas, e dividida em quatro
captulos: exerccios de sistemtica e representao, decifrando
enigmas (uma homenagem a Malba Tahan), problemas olmpicos
e raciocnio lgico.

Ali Iezid Izz-Eduim Ibn Salim


Hank Malba Tahan (18951974) era o pseudnimo de
Jlio Csar de Melo e Souza,
escritor e matemtico
carioca. Foi um dos maiores
incentivadores do ensino de
matemtica no Brasil.
Fonte: Wikipdia, a
enciclopdia livre.

O primeiro captulo, que d nome disciplina, apresenta problemas que sustentam os outros captulos, uma vez que sua resoluo obriga voc a organizar recursos intelectuais (sistemtica).
Ou, ento, estimula-o a buscar alguma representao grfica, algum recurso que transforme palavras escritas em conceito palpvel. Introduzimos rvores grficas para representar informao, o
que uma forma de representao clara e precisa e no ambgua
para problemas de natureza combinatria.
O segundo captulo uma seleo de problemas lgicos, enigmas para serem decifrados. H textos maravilhosos retirados ou
adaptados de Alice no pas das maravilhas, de Lewis Carroll,
ou de O homem que calculava, de Malba Tahan. H, tambm,
problemas que dependem da compreenso dos conectivos lgicos
e, ou, se ... ento, que so resolvidos com alguma sistemtica
e representao.
O terceiro captulo comea com trs provas olmpicas, que foram
as provas da primeira fase da OBM de 2004 nos nveis I, II e III.
O captulo apresenta, na seqncia, problemas, com resoluo, de
outras olimpadas, alm da brasileira: regionais, ibero-americanas, cone sul etc.
O quarto e ltimo captulo introduz o raciocnio lgico de modo
informal, mas rigoroso. Reciclando frases de publicidade, dilogos de filmes e enunciados matemticos, estudamos o clculo de
predicados, com ou sem quantificadores.

Esta disciplina envolve trabalho cuidadoso e leitura crtica. Convidamos voc a ler livros de Malba Tahan, de Martin Gardner, de
Lewis Carroll e todos os livros contidos na bibliografia. Esperamos que essa disciplina realmente lhe propicie condies de ser
mais sistemtico na abordagem de problemas matemticos.

Licio Hernanes Bezerra


Carmem S. Comitre Gimenez
Nereu Estanislau Burin
Martin Gardner

Lewis Carroll o
pseudnimo adotado pelo
matemtico e escritor
Charles Lutwidge Dodson
(1832-1898).

Introduo
H, pelo menos, duas dcadas que constatamos no Ensino Superior uma dificuldade crescente dos alunos em entender conceitos
matemticos, drasticamente entre alunos de cincias exatas e de
engenharias. Tornou-se quase dolorosa a transio do Ensino Mdio para o Ensino Superior. Um dos motivos parece ser a pouca
carga horria semanal de matemtica no Ensino Mdio, que no
consegue suprir as necessidades cognitivas para a aprendizagem
da matemtica superior - geralmente, disciplinas de clculo real,
clculo vetorial e lgebra linear.
Um outro motivo parece ser a relao entre estudante e professor,
que muda radicalmente de uma relao quase familiar (tia, tio)
para uma relao basicamente profissional. O Ensino Superior
propicia a capacitao do aluno em alguma profisso. Para isso,
porm, o aluno deve-se posicionar como um aprendiz com concentrao e disciplina suficientes para escutar, refletir, perguntar
e responder. A universidade fundamentalmente um laboratrio
onde o estudante ensaia para tornar-se suficientemente hbil a
fim de responder s demandas de sua vida profissional. Por isso,
responsabilidade (habilidade para responder) cobrada na universidade diferentemente do que cobrado no Ensino Mdio.
As dificuldades dos alunos para cursar disciplinas de matemtica
superior no so exclusivas de nosso pas, como se pode verificar
em stios de universidades europias na rede mundial de computadores, que oferecem cursos introdutrios ao clculo e lgebra
linear no perodo de frias imediatamente anterior ao comeo das
aulas. No curso de Licenciatura em Matemtica da Universidade
Federal de Santa Catarina (UFSC), algumas disciplinas iniciais
foram estruturadas para processar contedos de matemtica de
Ensino Fundamental e Mdio, mas com uma viso de matemtica superior, introduzindo uma maneira fundamentalmente matemtica de olhar, mesmo para as questes mais simples. Esta disciplina est includa entre elas.
H mais de uma dcada que o nosso curso de Licenciatura de Matemtica enfrenta essa realidade da transio. Poderamos apenas
lamentar o descrdito crescente da classe dos professores de En-

sino Fundamental e Mdio junto a sucessivos governos, ou pior, a


desmoralizao da educao por parte do sistema produtivo que
superdimensiona a remunerao de analfabetos funcionais, como
alguns jogadores de futebol, e paga uma ninharia a agentes de
sade que cuidam de doentes terminais, a professores que lidam
com nossas crianas e adolescentes etc. Em vez disso, resolvemos
enfrentar a situao como educadores.
Quais so as deficincias dos alunos? Ou melhor, o que ns, professores, percebemos nos alunos como obstculos para a compreenso de conceitos matemticos? Eis algumas observaes:
Os alunos no sabem seguir instrues recorrentes. Por
exemplo, eles geralmente no sabem ler um manual, como o
de imposto de renda. Em matemtica, h muitos problemas
em que voc tem que ler seguidamente as instrues para
saber se voc j considerou todas as possibilidades.
No usam representao alguma para um problema: uma
figura, um grfico etc.
Lem pouco e tm redao fragmentada.
No desenvolvem quase nenhuma abstrao de alto nvel,
em que os objetos no se comportam como algo conhecido e
seguem regras prprias (quase toda a matemtica assim).
a c ad + bc
Por isso, torna-se difcil entender por que + =
,
b d
bd
6
6
x +y
ou por que, em geral, 2
x3 + y 3 .
2
x +y
No particularizam conceitos e no generalizam resultados
corretamente.
So rgidos, desmotivados e, muitas vezes, arrogantemente
prepotentes. Ser que essa arrogncia do aluno de hoje no
resultado da ausncia de limites para o aluno na escola?
H vrios casos de pais que minam a autoridade do professor na escola, por exemplo, favorecendo-o sempre em querelas entre professor e aluno. claro que tambm h abusos
por parte de professores. Mas, aqui, perguntamos: a famlia
assiste as suas crianas na escola de uma forma satisfatria? Porque isso extrapola at a universidade. O estudante

Inclui todas as pessoas que


possuem menos de quatro
sries de estudo concludos.
Estas, reconhecem letras
e palavras, mas no
interpretam textos.

Grafo
Conjunto formado por pontos (chamados ns ou vrtices) e por linhas (arestas) que
ligam alguns desses pontos.
O grafo pode ser orientado
(neste caso, a ligao entre
pontos A e B diferente de
ligao entre B e A), ou no.
Exemplo: rvores.

Aps a 2 Guerra Mundial, a


cidade da Prssia ocidental,
Knisberg, passou a ser
uma cidade russa e mudou
de nome para Kaliningrad.
Esta cidade possui um rio
com duas ilhas conectadas
por sete pontes. O problema
era saber se era possvel
caminhar de um ponto
qualquer da cidade e
retornar a este ponto
passando por cada ponte
exatamente uma vez.
Nasceu em So Paulo
(1931). Poeta, tradutor,
ensasta, crtico de
literatura e msica. A
partir de 1980, intensificou
os experimentos com as
novas mdias, apresentando
seus poemas em placas
luminosas, videotextos,
neon, hologramas
e laser, animaes
computadorizadas e eventos
multimdia, abrangendo
som e msica. Fonte:
www.uol.com.br/
augustodecampos

Em que se exploram frases


do tipo: se voc se encaixa
em um perfil bvio ento
voc vai comprar o produto.

prepotente prejudica a si mesmo: em vez de abrir a mente


para algo novo, ele se prende em um atoleiro emocional. Um
exemplo disso o fato comum de alunos atriburem apenas
ao professor de uma disciplina a culpa da sua reprovao.
Esta disciplina apresenta as seguintes propostas:
a) Introduzir formas de representao de dados, como smbolos, tabelas e grafos. Para isso, elaboramos problemas ou
adaptamos exerccios de vrios livros (cujas referncias aparecem no final do livro) que, para resolv-los, seja inevitvel
uma representao. Aproveitamos a ocasio para introduzila. Exemplos: quadrado mgico 3x3, problemas de possibilidades, o problema da ponte de Knisberg, problemas de
percursos mnimos etc.
b) Submeter o estudante a exerccios em que seja obrigatria
a leitura incessante de uma lista de instrues. Exemplo: a
formao de quadrados mgicos mpares, de ordem > 3.
c) Envolver o aluno com problemas em que seja necessrio o
uso de uma sistemtica para resolv-los. Exemplo: problemas de mximo divisor comum entre alguns inteiros, problemas de abrir porta com vrios ferrolhos, cada um abrindo em sentidos diversos etc.
d) Expor o aluno a problemas ldicos, que surgem em diversas
referncias literrias. Exemplo: doublets, de Lewis Carroll;
triplets, de Augusto de Campos; o problema das escravas
com burcas, de Malba Tahan.
e) Desafiar o aluno com problemas olmpicos, utilizados em
olimpadas regionais, brasileiras, ibero-americanas, internacionais de matemtica.
f) Introduzir o raciocnio dedutivo, que inerente matemtica, via frases oriundas de publicidade, teoremas de geometria euclidiana etc.
Pretendemos, com esses problemas, que o estudante se envolva
com a matemtica com mais prazer e disponibilidade, que ele
descubra o fato que, quando est pensando em um problema de
matemtica, ele est conectado com toda uma linhagem de ma-

temticos do passado e do presente. Que o estudante encare essa


maravilha e esse mistrio com respeito e dignidade, honrando
assim a tradio da matemtica.

Captulo 1
Exerccios de Sistemtica
e Representao

15

Captulo 1
Exerccios de Sistemtica e
Representao
1.1 Exerccios de Sistemtica
Para entender o que sistemtica, vamos apresent-la em um
exemplo famoso, a construo do quadrado mgico 3x3, que
nico a menos de simetrias e/ou rotaes.

O quadrado dito mgico


porque a soma dos nmeros
de qualquer linha, coluna
ou diagonal, se mantm
constante.

Albrecht Drer (1471-1528)


Gravador e pintor alemo.
Grande terico da cidade
do Renascimento. Publicou,
em 1527, o Tratado sobre
fortificao de cidades, vilas
e castelos, onde apresenta
um esquema de uma cidade
ideal quadrada. O texto
referido como obraprima para arquitetos e
urbanistas. Fonte: Wikipdia,
enciclopdia livre.

O problema o seguinte: construa uma matriz 3x3 com os nmeros de 1 a 9, de tal modo que a soma das entradas de qualquer
linha ou coluna ou diagonal (principal e secundria) seja sempre
a mesma.
A origem dos quadrados mgicos parece situar-se na China, atribuda ao imperador engenheiro Yu, o Grande, que reinou por volta de 2.200 a.C. O quadrado mgico 3x3, conhecido como Lo Shu,
associado ao segundo arranjo dos trigramas do I Ching, que
denominado Cu Posterior ou Wenwang, por ser atribudo ao Rei
Wn, que reinou em torno de 1.100 a.C. Conta a lenda que esse
quadrado foi visto por Yu no dorso de uma tartaruga que surgiu
do rio Lo. Entretanto, foi s no sculo IX d.C. que os quadrados
mgicos propagaram-se pelo Japo e Oriente Mdio, chegando
ndia no sculo XI. Quadrados mgicos gravados em placas de
prata foram usados como amuleto, na Europa medieval, contra a
peste. Na gravura 1.2, Melancholia, de Albrecht Drer, do sculo
XVI, aparece um quadrado mgico de ordem 4, com soma 34, entre outras referncias alquimia e astrologia.

Figura 1.1 - Evoluo do Quadrado Mgico

16

Figura 1.2 - Melancholia

A primeira pergunta que uma mente sistemtica faz : qual o


valor dessa soma?
Bem, a soma de todos os nmeros de 1 a 9 45. Na matriz, esses
nmeros sero arranjados de tal modo que a soma na primeira linha
ser igual soma na segunda linha, que ser igual, ainda, soma na
terceira linha. Ou seja, teremos 3 somas iguais, cuja adio resultar
em 45. Ou seja, a soma procurada 15.
Uma outra pergunta : de quantos modos podemos escrever 15
como uma soma de trs parcelas distintas, escolhidas entre os nmeros de 1 a 9?
O maior nmero 9. Se eu escolher somar outros dois nmeros com
ele para dar 15, o prximo nmero no mximo 5 e o terceiro, ento,
1.
9 + 5 + 1 = 15
Ainda com 9 como primeira parcela, temos
9 + 4 + 2 = 15
Note que na prxima soma apareceria o nmero 3 duas vezes, o que
foge regra.

17

Agora com 8 como primeira parcela, temos ento:


8 + 6 + 1 = 15
8 + 5 + 2 = 15
8 + 4 + 3 = 15
Com 7,
7 + 6 + 2 = 15
7 + 5 + 3 = 15
Com 6,
6 + 5 + 4 = 15
Podemos olhar, agora, para a matriz quadrada vazia e notar que a
entrada no centro da matriz participa de quatro somas: as das duas
diagonais, mais a da linha e a da coluna que se cruzam no centro.
As entradas relativas aos quatro vrtices do quadrado participam,
cada uma, de trs somas: a da linha e a da coluna que se juntam
no vrtice, mais a soma da diagonal. Finalmente, as outras quatro
entradas participam apenas de duas somas.
Observando a lista de somas acima, conclumos que 5 est no centro, pois o nico nmero que participa de quatro somas. Um dos
vrtices 6 (que participa de trs somas) e, logo, o oposto a ele 4
(idem). Os outros dois vrtices so 8 e 2 (idem). Ento se completa a
matriz com os outros quatro nmeros.
15

15

15

15

15

15

15
15

Figura 1.3 - Lo Shu quadrado mgico 3x3.

18

Um dado frustrante que essa sistemtica no resolve completamente o problema para quadrados mgicos de ordem maior, por
exemplo, um quadrado mgico 4x4.
Vimos que existe, a menos de rotaes e simetrias, apenas um quadrado mgico 3x3. Porm, existem 880 quadrados mgicos 4x4 e
275.305.224 quadrados mgicos 5x5 (sempre a menos de simetrias e
rotaes). O nmero de quadrados mgicos 6x6 ainda desconhecido, mas estimado ser na ordem de 1019.
Os quadrados mgicos pares (de ordem maior ou igual a 4) tm resoluo complicada. Uma explanao sobre quadrados mgicos e
como obt-los pode ser vista no excelente stio de matemtica mathworld.wolfram.com, um dos mais completos no mundo. Quadrados mgicos esto em mathworld.wolfram.com/MagicSquare.html.
Um exerccio interessante gerar quadrados mgicos de ordem mpar seguindo as seguintes instrues, que foram descritas por Kraitchik (KRAITCHIK, M. Mathematical Recreations. New York: Norton,
1942):
coloque o nmero 1 no centro da primeria linha (a de cima);
c oloque cada nmero subseqente no quadrado imediatamente direita e acima;

Esse mtodo conhecido


como Mtodo Siams ou
Mtodo de la Lourbre.
Parece que a primeira vez
que esse mtodo surgiu
no ocidente foi no sculo
XVII, quando Antoine de
la Lourbre retornou
Frana aps servir no Sio
(hoje, Tailndia), como
embaixador.

s e sair do quadrado ao ir para a direita, o quadrado subseqente o da mesma linha (de cima), na primeira coluna (a
da esquerda); se sair do quadrado ao ir para cima, o quadrado
subseqente ser o da mesma coluna (da direita), na ltima
linha (a de baixo);
q
uando encontrar, nesse processo, alguma casa ocupada, o
prximo nmero ser posto imediatamente abaixo do nmero
anterior (o que foi colocado por ltimo).
Obs.: Esse movimento em diagonal fica claro se considerarmos quadrados gmeos, colocando o nmero na posio correspondente ao
quadrado original.
Se comearmos o processo com o nmero 1 em qualquer casa, esse
mtodo ainda gera um quadrado mgico fraco, isto , um quadrado

Quadrados gmeos so
quadrados idnticos postos
volta do quadrado
original; ao todo so oito:
um de cada lado, um em
cada vrtice.

19

em que a soma dos elementos das linhas, das colunas, e de apenas


uma das diagonais sempre a mesma. Verifique e calcule quantos
quadrados mgicos podemos gerar por esse procedimento, a menos
de simetrias e rotaes.

1.1.1 Exerccios Propostos


1) Construa quadrados mgicos 3 por 3, com os seguintes nmeros (sem repetir):
a) de 2 a 10
b) os pares de 2 a 18
c) os mpares de 1 a 17
2) Disponha as culturas de milho, arroz, feijo e soja - quatro de
cada uma - num campo quadrado 4 por 4 de modo que cada
cultura s aparea uma vez em cada linha, coluna ou diagonal.
Sugesto: fixe alguma linha, coluna ou diagonal. Depois de fixada, quantas solues existem?
3) Uma porta tem 3 trincos e voc no sabe para que lado cada
trinco abre. O problema abrir a porta. Note que:
o problema tem uma nica soluo
se voc sair tentando ao acaso, voc pode dar o azar de nunca chegar soluo
se voc for cuidadoso, na pior das hipteses vai precisar de 8
tentativas.
Quais so as 8 possibilidades? Descubra um mtodo para girar
os trincos que as simule.
4) Considere a seguinte formao de nmeros, construdos com
os algarismos de 0 a 9, em que os dgitos so considerados da
esquerda para a direita:
i) os algarismos do nmero so todos diferentes;
ii) cada algarismo resultado de uma das seguintes operaes:
ou ele trs vezes o anterior, ou uma unidade a mais que
isso, ou duas unidades a mais que isso;

20

iii) se uma das operaes resultar em um nmero maior que 9,


considere somente o algarismo das unidades do resultado;
iv) para o primeiro algarismo, o algarismo anterior o ltimo
algarismo;
v) o primeiro algarismo o maior algarismo do nmero.
Determine todos os nmeros construdos desta forma,
a) com 1 algarismo;
b) com 2 algarismos;
c) com 3 algarismos.
5) Um jogo de domin que vai at o duplo 6 possui 28 peas.Um
jogo de domin que vai at o duplo 9 possui 55 peas. Quantas
peas possui um jogo de domin que vai at o duplo 12? Voc
pode generalizar?
6) Quantos tringulos so possveis de se formar com os vrtices
quaisquer de um pentgono regular? E com os de um hexgono regular? E com os de um octgono regular?
7) Num prdio de 6 andares (sem contar o trreo) as escadas
de um andar para o outro tm todas o mesmo comprimento.
Quantas vezes a escada do 1 ao 6 andar mais alta do que a
escada do 1 ao 3? Qual a razo entre o comprimento das escadas do 1 ao 6 e o comprimento das escadas do 1 ao 3?
8) Temos 30 peas no formato de tringulo eqiltero e gostaramos de formar o maior hexgono possvel com estas peas,
sem espaos vazios no seu interior.
a) Quantas peas no sero usadas?
b) Se o lado do tringulo mede 1 unidade, qual a medida do
lado do hexgono?
9) Cinco corvos esto lado a lado e igualmente afastados sobre
uma cerca. Tente descobrir em que ordem os corvos esto, a
partir das seguintes pistas:

21

a) C est mesma distncia de A que de B.


b) E est entre D e A.
c) B est ao lado de E.
d) E no est entre B e D.
10) Qual o menor nmero que tem a propriedade de ficar com
seus dgitos invertidos quando multiplicado por 9?
11) Por qual nmero deve-se multiplicar 49 para obter 4.949? Por
qual nmero deve-se multiplicar 38 para obter 383.838? Generalize para nmeros com 2 dgitos.
12) Um livro tem 500 pginas. Quantas vezes o algarismo 1 aparece na numerao?
13) Quantos nmeros naturais com 4 algarismos diferentes existem tal que a diferena entre o penltimo algarismo e o ltimo algarismo em valor absoluto seja 2?
14) Qual a soma de todos os inteiros entre 50 e 350 que terminam em 1?
4 de Julho o 185 dia
do ano no calendrio
gregoriano (186 em anos
bissextos). Faltam 180 para
acabar o ano. Chama-se
ano bissexto o ano que
possui um dia a mais do que
os anos comuns. O objetivo
manter o calendrio
utilizado em sincronia
com os eventos sazonais
relacionados s estaes
do ano. Fonte: Wikipdia, a
enciclopdia livre.

15) Na Amrica do Norte uma data como 4 de julho de 1999 escreve-se geralmente como 7/4/99, mas em nosso pas o ms
aparece em segundo lugar e a data ser escrita como 4/7/99.
Se no soubermos que sistema utilizado, quantas datas so
ambguas em relao a essas notaes?
16) s 6 horas o relgio da igreja levou 30 segundos para dar as
6 badaladas. Jorge concluiu que, ao meio-dia, ele levaria 1 minuto para dar as 12 badaladas. Mas ele estava enganado. Qual
o tempo correto?
17) Uma pessoa faz 69 anos em 96, 58 anos em 85, 47 anos em 74
etc. Uma outra pessoa faz 79 anos em 97, 68 anos em 86, 57
anos em 75 etc. claro que esta coincidncia no acontece com
todas as pessoas. Qual a condio para que isso ocorra?

22

18) Um grupo de fanticos religiosos decidiu, aps um intenso


estudo dos seus livros sagrados e uma imensa utilizao de
poderosos computadores, que o fim do mundo aconteceria
quando o primeiro dia de um sculo vindouro casse num
domingo. Segundo eles, quanto tempo ainda nos resta?
19) Com uma ampulheta que marca 7 minutos e outra que marca
11 minutos, como que se pode marcar o tempo de 15 minutos para cozinhar um ovo?
20) Quatro amigas partilham um apartamento. Enquanto uma
delas prepara caf, outra estuda matemtica, outra estuda literatura e a outra l jornal. Descubra o que cada uma est fazendo se:
i) Marta no est estudando matemtica e no est lendo jornal.
ii) Maria no est preparando o caf e no est estudando matemtica.
iii) Se Marta no est preparando o caf, ento Mirna no est
estudando matemtica.
iv) Mrcia no est lendo jornal e no est estudando matemtica.
v) Mirna no est lendo jornal e no est preparando o caf.
21) Mil armrios esto enfileirados e numerados de 1 a 1.000. Mil
alunos, tambm numerados de 1 a 1.000, comeam a seguinte
brincadeira:
i) o primeiro aluno abre as portas de todos os armrios (que,
inicialmente, estavam fechados);
ii) o aluno 2 inverte a situao das portas 2, 4, 6, 8, (isto , fechaas);
iii) o aluno 3 inverte a situao das portas 3, 6, 9, 12, (abre-as ou
fecha-as).
Sucessivamente, cada aluno passa e inverte as situaes das
portas dos armrios que tm nmeros mltiplos de seu prprio
nmero. Aps os mil alunos passarem, qual o nmero do ltimo armrio a permanecer aberto? Quantos armrios permanecero abertos? Quais?

23

1.1.2 Soluo dos Exerccios Propostos


2) Fixada uma linha, coluna ou diagonal, temos duas solues.
4) Para determinar quais nmeros de apenas um algarismo satisfazem as condies, usam-se as instrues (iv) e (ii). Comecemos pelo 9: considerando-o como primeiro e ltimo algarismo,
verificaremos se a instruo (ii) ser satisfeita. Agora, 9 3 = 27,
9 3 + 1 = 28, 9 3 + 2 = 29 e os algarismos considerados seriam 7, 8 e 9. Assim, o nmero 9 satisfaz essa instruo. Analogamente, pode-se verificar que 4 e 5 satisfazem todas as condies listadas.
Para nmeros de dois algarismos, observando a instruo (v),
vamos iniciar tambm por nmeros cujo primeiro algarismo
9: 9 3 = 27; as possibilidades para o segundo algarismo sero
7 ou 8 (o prprio 9 no serve pela instruo (i)). Agora, como
7 3 = 21, 7 3 + 1 = 22 e 7 3 + 2 = 23, o nmero 7 no pode
ser considerado um algarismo anterior a 9.
Do mesmo modo, no se pode considerar 8 como algarismo
anterior ao 9, pois 8 3 = 24 , 8 3 + 1 = 25, 8 3 + 2 = 26.
Assim, no teremos nenhum nmero de dois algarismos comeando com 9 que satisfaa todas as instrues.
Usando o mesmo procedimento, comeando agora por 8, teremos 86; com 7, 72; com 3, 31.
Use o mesmo procedimento para encontrar todos os nmeros
de trs, quatro e cinco algarismos. Se tudo correr bem, voc
encontrar 6 nmeros de trs algarismos, 8 nmeros de quatro
algarismos e 7 nmeros de cinco algarismos.
5) 91 peas.
6) Temos 5 vrtices no pentgono. Logo, temos 5 maneiras de escolher o primeiro vrtice do tringulo, 4 maneiras de escolher
o segundo vrtice e 3 maneiras de escolher o terceiro. No entanto, esses tringulos esto sendo contados seis vezes. Pois, se
A, B, C, D e E so os vrtices do pentgono, os tringulos ABC,
ACB, BAC, BCA, CAB, CBA so todos o mesmo tringulo! Assim, vemos que 60 (= 5 4 3) o nmero de tringulos com
repeties. Como cada tringulo pode ser representado de 6

24

maneiras diferentes, devemos dividir 60 por 6, obtendo, assim,


um total de 10 tringulos diferentes. Usando o mesmo raciocnio, poderemos formar 20 tringulos diferentes com os vrtices de um hexgono e 56 tringulos diferentes com os vrtices
de um octgono.
7) 2 vezes e meia.
8) O maior hexgono formado com 24 peas; seu lado mede 2
unidades (o dobro do lado do tringulo); sobram, portanto, 6
peas. Faa o mesmo problema usando 2.002 peas.
9) No possvel descobrir a ordem em que esto os corvos na
cerca. As pistas so contraditrias; se comearmos a disposio
com as pistas 1 e 2, teremos
A-------C-------B ou B-------C-------A.
Considerando a primeira opo, a pista 2 nos levaria a
D-------E-------A-------C-------B,
o que contradiz as pistas 3 e 4. Analogamente, acontecem contradies se comearmos com as pistas 3 ou 4. Assim, a resposta : o problema no tem soluo.
10) 1.089.
11) 49 101 = 4.949; 38 10 101 = 383.838.
12) 200 vezes
13) 798 nmeros
14) A soma 5.880.
15) 132 datas ambguas por ano.
16) O tempo correto 66 segundos.

25

17) A condio para que isso ocorra a pessoa ter nascido num
ano cujos dois ltimos algarismos formam um nmero mltiplo de 9.
Gregrio XIII, nascido Ugo
Buoncampagno (1502
1585), destaca-se por ter
sido o responsvel pela
introduo do calendrio
atual (gregoriano),
reformando o antigo
calendrio juliano.

18) Em 1582, o Papa Gregrio XIII decretou que, ao invs de haver


um ano bissexto a cada 4 anos, seria feita uma exceo para
todos os anos que fossem mltiplos de 100 e no fossem mltiplos de 400. Isto ajustaria as horas que vo se acumulando
com o passar dos anos, apesar do acerto dos anos bissextos.
Lembre-se que um ano tem 365 dias, 6 horas, 13 minutos e 53
segundos. Por exemplo, o ano 2000 foi um ano bissexto (mltiplo de 4 e de 400), mas o ano 2100 no ser bissexto, apesar
de ser mltiplo de 4, pois no divisvel por 400. Sabendo
disso, podemos fazer a contagem e verificar que o primeiro
dia de um sculo nunca cair num domingo.
19) D-se a partida nas duas ampulhetas. Quando a ampulheta
de 7 minutos se esgotar, pe-se o ovo na gua fervente. Quatro minutos depois a ampulheta de 11 minutos tambm se
esgota. Basta vir-la de novo e, quando tiver se esgotado pela
segunda vez, o ovo ter cozinhado por 15 minutos.
20) Podemos usar um quadro 4 por 4 e eliminar algumas possibilidades:
Estuda matemtica
Estuda literatura
Prepara caf
L jornal

Marta

Maria

Mirna

Mrcia

N
S

S
N

N
S

N
N

As instrues (i), (ii), (iv) e (v) eliminam duas clulas cada


uma. A instruo (iii) uma implicao. Declara que, se Marta
no est preparando o caf, ento Mirna no est estudando
matemtica. Observemos, porm, que, como cada garota est
fazendo apenas uma coisa, s resta a Mirna estar estudando
matemtica, e a Maria estar lendo jornal. Assim, se chamarmos Marta no est preparando o caf de A e Mirna no
est estudando matemtica de B, teremos (A B) e ~B (a negao de B, j que Mirna est estudando matemtica), o que

26

nos d, como concluso, a negao de A, ou seja, Marta est


preparando caf (ver captulo 4). Isto implica que Mrcia est
estudando literatura. A soluo ento: Marta est preparando o caf, Maria est lendo jornal, Mirna est estudando matemtica e Mrcia est estudando literatura.
21) Os armrios que permanecem abertos so aqueles que tm
sua posio invertida um nmero mpar de vezes. O nmero
de vezes que a posio de um armrio n muda a quantidade de divisores do nmero n. Para calcular essa quantidade,
decompe-se n em fatores primos, na forma n = p1 . p2 . ... . pk .
A frmula que nos d d(n) a quantidade de divisores de um
nmero n :
d(n) = (s1 + 1).(s2 + 1). ... .(sk + 1).
Para que este produto seja mpar, todos seus fatores devem ser
mpares, ou seja, os nmeros s1, ... , sk devem ser pares. Como
esses nmeros correspondem aos expoentes dos primos p1, ... ,
pk , da decomposio de n, este nmero dever ser um quadrado
perfeito. Assim, os armrios que permanecero abertos sero
aqueles cujos nmeros so quadrados perfeitos. Finalmente, o
ltimo armrio aberto corresponde ao maior quadrado perfeito menor que 1.000, que 961 = 312.

1.2 Exerccios de Representao


Nesta seo, abordamos um tipo de problema tal que sua soluo se
apresente de forma bvia, uma vez que alguma forma de representao dos seus dados utilizada. Por exemplo, o problema de descobrir qual o valor de um objeto sabendo-se que o triplo da quantia
paga por ele o troco recebido pelo seu pagamento com uma nota
de cem reais. Representamos a quantia paga por uma varivel, em
geral x, e escrevemos, ento, 3x = 100 x. Resolvendo essa igualdade
(equao), temos que a quantia paga foi 25 reais.
Muitos dos problemas aqui propostos podem ser resolvidos com o
auxlio de um grafo especial as rvores (como uma rvore genealgica). Essa forma de representar dados eficiente, pois nela os
dados so aparentes e fceis de serem verificados. Resolva o primei-

rvore Genealgica
Representao grfica dos
antepassados de um indivduo, que pode incluir tambm, em cada gerao, os
parentes colaterais.
Fonte: Dicionrio Houaiss

27

ro exerccio proposto. Tente, ento, resolver o segundo exerccio, de


algum modo. Verifique, ento, a resoluo desse exerccio em 1.2.2,
onde voc encontrar mais informaes sobre rvores.

1.2.1 Exerccios Propostos


1) Construa a rvore genealgica dos descendentes de seu av
paterno, seguindo as instrues:
a) a partir dele, desa uma linha a cada gerao nova
b) entre quaisquer dois irmos represente o mais velho esquerda do mais novo
Supondo que a diferena mnima entre duas geraes seja de
20 anos e entre dois irmos seja de 1 ano, estime, a partir de
sua idade, a idade de seu av.
2) Quantas so todas as filas de 4 smbolos diferentes com as letras A, B, C, D? Quais so? Como voc garante que todas foram
listadas?
3) Quantos nmeros diferentes de 10 algarismos distintos podem-se formar com os 10 dgitos? Lembre-se que nmeros no
comeam com zero.
4) Quantos nmeros com 3 dgitos, diferentes entre si, podemos
escrever com os algarismos 0, 1, 2, 3, 4? Quantos so os nmeros pares? Quantos so os mpares?
5) O alfabeto usado no planeta Y tem somente duas letras: W e
K. O sobrenome de cada um de seus habitantes uma seqncia formada por 4 letras. Por exemplo, WKWW um possvel
sobrenome usado nesse planeta. Qual o maior nmero de
sobrenomes diferentes que podem ser criados no planeta Y?
6) Chamam-se palndromos os nmeros naturais que no se alteram quando a ordem dos algarismos invertida como, por
exemplo, 2002, 121, 12321 etc. Quantos palndromos de 3 algarismos existem?

28

7) Queremos pintar as faces de um paraleleppedo e dispomos de


5 cores: azul, verde, vermelho, preto e branco. Queremos que
faces opostas tenham a mesma cor e faces contguas tenham
cores diferentes. De quantas maneiras podemos fazer isso?
8) Reunindo-se 5 quadrados iguais, lado com lado, de todas as
maneiras possveis, encontram-se 12 figuras distintas. Descreva essas 12 figuras e arranje-as de forma a obter um retngulo
(essas 12 figuras so conhecidas como pentamins).
9) Hexamins so figuras formadas por 6 quadrados iguais, da
mesma forma que os pentamins do exerccio anterior. Quantos hexamins diferentes existem ? Quais se podem dobrar
para formar um cubo?
10) Uma partio de um nmero inteiro positivo n uma diviso
em pedaos inteiros e positivos (logo no nulos) de n. Por
exemplo: uma partio de 7 seria 3 + 2 + 2, outra seria 4 + 2 +
1. Numa partio no levamos em conta a ordem das parcelas,
uma vez que a adio comutativa. Assim, 3 + 3 + 1 e 3 + 1 +
3 seriam a mesma partio. Obtenha todas as parties de 5,
6 e 7.
11) Um nmero chamado bom se admite uma partio cuja
soma dos inversos igual a 1. Liste todos os nmeros bons
de 1 algarismo. Liste 3 nmeros bons de 2 algarismos.
12) Em 1971 o Sr. Nunes disse a um amigo: houve um ano que era
exatamente o quadrado da minha idade naquele ano. Em que
ano nasceu o Sr. Nunes?
13) Disponha bolas em forma de tringulo eqiltero, com n bolas
na base, n 1 bolas na fileira de cima, n -2 bolas na prxima
etc., at terminar com uma bola. O nmero total de bolas no
tringulo ser a soma dos nmeros de 1 a n. Os nmeros assim
obtidos so chamados nmeros triangulares. De modo anlogo, empilhe bolas em formato de pirmide de base triangular
(tringulo eqiltero); chamaremos o total de bolas neste caso
de nmeros tetradricos. Encontre uma frmula para os nmeros triangulares e determine os primeiros doze. Encontre

29

uma frmula para os nmeros tetradricos e calcule, tambm,


os primeiros doze. Que relao existe entre os nmeros triangulares e os nmeros quadrados?
14) O nmero 36 um nmero triangular e tambm um quadrado perfeito; o menor nmero acima de 1 com esta caracterstica. O segundo nmero nestas condies 1.225. Quais so os
prximos dois nmeros?

1.2.2 Soluo dos Exerccios Propostos


2) Uma maneira de organizar e representar os dados desse problema com o uso de uma rvore, que um tipo de grafo, cuja
teoria foi iniciada por Euler, ao resolver o clebre problema das
7 pontes de Knigsberg.

Euler
Matemtico e fsico,
Leonhard Euler resolveu
enorme quantidade de
problemas, da navegao
s finanas, da acstica
irrigao.

Diz-se que o problema matemtico das 7 pontes de Knigsberg foi muito popular na Knigsberg do sc. XVIII, que
uma cidade conhecida, hoje, como Kaliningrado, um enclave da Rssia no mar Bltico. O desafio era resolver se existia
ou no um caminho que percorresse todas as sete pontes,
passando exatamente por elas uma nica vez. Esse problema pode ser solucionado, em princpio, testando-se sistematicamente todas as possibilidades. Entretanto, o modo
Euler resolveu o problema considerado, hoje em dia, como
a origem da atual Teoria de Grafos, um ramo da matemtica que se vem mostrando eficiente em modelao de redes,
processos de produo, processos logsticos etc. Uma resoluo desse problema est em:
www.inf.ufsc.br/grafos/temas/euleriano/konigsbgerg.htm

Figura 1.4 - rvore

Essa forma de representao de dados garante que voc realmente represente todas as possibilidades para o problema. Comeamos escolhendo uma letra para iniciar a fila, digamos, a
letra A (figura 1.4). Partem desta letra (um n ou vrtice da rvore) segmentos de reta que simulam os ramos da rvore (arestas), que terminam em cada uma das 3 possibilidades para a
segunda letra da fila, que deve ser diferente da primeira letra,
ou seja, B, C ou D. De cada um desses ns saem novas arestas

30

ligando-os s possibilidades para a terceira letra. Por exemplo,


do n B saem duas arestas para os ns C e D. De cada um dos
ns correspondentes terceira letra, sai mais uma aresta que
ligar o n a apenas um n: a quarta letra.
Teremos, ento, exatamente 6 filas de 4 letras diferentes, comeando com a letra A (estas so todas as filas possveis). Um
procedimento anlogo pode ser feito para a contagem das filas
comeando com as letras B, C e D - 6 filas em cada caso. Assim,
no total, h 6 4 = 24 filas. As trilhas que comeam no primeiro
n e terminam no quarto n descrevem quais so essas filas.
3) Podem-se formar 3.265.920 nmeros.
4) 48 nmeros com dgitos diferentes; 30 pares e 18 mpares.
Sugesto: resolva o problema por rvores.
5) 16 sobrenomes diferentes (sugesto: use rvores).
6) 90 palndromos.
7) 60 maneiras (sugesto: use rvores).
8) Os 12 pentamins esto assinalados na figura abaixo. Voc
pode construir outros retngulos com eles?

9) Existem 35 hexamins. 11 podem ser dobrados para formar


cubos.
10) 6 parties de 5, 10 parties de 6 e 14 parties de 7.
11) So nmeros bons de um algarismo o 1, o 4 e o 9. Outros exemplos: 16, 25, 64. Observe que todos os quadrados perfeitos so
bons. Por qu? D mais exemplos que no sejam quadrados.

31

12) O Sr. Nunes tinha 44 anos em 1936, que igual a 442. Ele nasceu em 1892.
13) Nmeros triangulares: Tn =

n(n + 1)
, que a soma dos termos
2

da progresso aritmtica 1, 2, 3, ....


T1 = 1, T2 = 3, T3 = 6, T4 = 10, , T12 = 78.
Nmeros tetradricos:
Pn =

k (k + 1) = 1 n 2 1 n
n(n + 1)(n + 2) .
k + k =


2
6
2 1
2 1

P1 = 1, P2 = 4, P3 = 10, P4 = 20, , P12 = 364.


Relao entre os triangulares e os quadrados: Qn = Tn-1 + Tn .
14) Os prximos nmeros so 41.616 (este o 288o nmero triangular, que igual a 2042) e 1.413.721 (este o 1.681o nmero
triangular, que igual a 1.1892).

Referncias
[1] BEZERRA, Licio H. et al. Introduo Matemtica. Florianpolis:
Ed. UFSC, 1995.
[2] BOLT, Brian. Atividades Matemticas. Lisboa: Gradiva, 1991.
[3] __________. Mais Atividades Matemticas. Lisboa: Gradiva, 1992.
[4] BOYER, Carl B. Histria da Matemtica. 2. ed. So Paulo: Edgard
Blcher, 1996.
[5] GARDNER, Martin. Divertimentos Matemticos. 4. ed. So Paulo:
Ibrasa, 1998.
[6] MALBA TAHAN. O homem que calculava. 56. ed. Rio de Janeiro:
Record, 2002.
[7] MARIANI, Antonio C. Teoria dos grafos. Disponvel em: <www.
inf.ufsc.br/grafos>. Acesso em: 29 mai. 2009.
[8] __________. Problemas das Pontes de Konigsberg. Disponvel em:
<www.inf.ufsc.br/grafos/problemas/konigsb.htm>. Acesso em:
01 jun. 2009.

Captulo 2
Decifrando Enigmas
uma homenagem a Malba
Tahan

35

Captulo 2
Decifrando Enigmas uma
homenagem a Malba Tahan
2.1 Introduo

Viveu 79 anos (de 1895 a


1974), a maior parte no Rio
de Janeiro. Em 1947, funda
as revistas de recreao
matemtica Al-Karisme
e Damio. Alm de obras
didticas, escreve romances
O Homem que Calculava
(1938) e O Livro de Aladim
(1943). Sobre a obra de
Malba Tahan, Monteiro
Lobato disse que ... ficar
a salvo das vassouradas
do tempo como a melhor
expresso do binmio
cincia-imaginao.
Fonte: Wikipdia, a
enciclopdia livre.

Problemas de natureza lgica enigmas, quebra-cabeas - sempre


foram utilizados na histria da humanidade como instrumento
de ensino. Mestres desafiavam seus discpulos com problemas
desse tipo para, principalmente, quebrar rotinas de raciocnio
estabelecidas pelos alunos. O choque criado pelo enunciado dos
problemas permitia que o crebro se esvaziasse por uns instantes
da torrente de pensamentos mecnicos, costumeiros, obtendo-se
assim algum espao para a compreenso de algo novo. So exerccios em que se exigem ateno e concentrao. Em geral, so
apresentados como um conto, uma pequena histria, de forma
inusitada, com o propsito de distrair a mente mecnica de seu
controle tacanho sobre a percepo do mundo, para que se desenvolva a intuio, verdadeiro pilar das descobertas humanas.
Na Inglaterra, o reverendo Charles Lutwidge Dodgson (nascido
em 27/01/1832, falecido em 1898), mais conhecido por Lewis Carroll, seu pseudnimo, dedicou-se a escrever livros em que a lgica
era discutida em dilogos de alguns de seus personagens. No Brasil, os livros de Jlio Csar de Mello e Souza, cujo pseudnimo era
Malba Tahan, continuam a interessar leitores de vrias geraes.
Os exerccios aqui apresentados seguem o estilo dos contidos nesses livros (alguns so adaptados a partir de trechos desses livros).

2.2 Enigmas, quebra-cabeas,


dobletes, tripletes
1) Uma costureira colocou todos os botes que tinha em trs
caixas: uma, s com botes brancos, que ela rotulou com B;
outra, s com botes pretos, que ela rotulou com P; e a ter-

36

ceira, com botes de vrias cores, mas todos diferentes de pretos e brancos, que ela rotulou com M. Um dia, seu neto mais
capeta trocou os rtulos das caixas, no deixando nenhuma
com o rtulo original. No entanto, retirando um nico boto
de uma das caixas, a costureira foi capaz de descobrir o que
cada uma continha. Como ela fez isso?
2) Natal e Pascoal, cada um acompanhado por um de seus filhos,
foram pescar. Natal pescou o dobro de peixes que pescou seu
filho, enquanto Pascoal pescou o quntuplo de peixes que seu
filho pescou. No total, pescaram 26 peixes. Leonardo o nome
do filho de Natal. Como se chama o filho de Pascoal e quantos
peixes ele pescou?
3) Alice, ao entrar na floresta, perdeu a noo dos dias da semana. O leo e o unicrnio eram duas estranhas criaturas que
freqentavam a floresta. O leo mentia nas segundas, teras e
quartas-feiras, e falava a verdade nos outros dias da semana. O
unicrnio mentia nas quintas, sextas e sbados, e falava a verdade nos outros dias da semana. Alice encontrou o leo e o unicrnio descansando sombra de uma rvore. Eles disseram:
Leo: Ontem foi um dos meus dias de mentir.
Unicrnio: Ontem foi um dos meus dias de mentir.
A partir destas informaes Alice descobriu qual era o dia da
semana. Qual era?
4) Numa estranha ilha do planeta Z, a cada dia da semana, cada
um dos habitantes ou mente o dia todo ou passa o dia todo
dizendo a verdade. Todos os habitantes podem mentir em certos dias e dizer a verdade em outros, mas no decorrer de um
mesmo dia da semana seu comportamento constante. Alm
disso, para cada habitante A, existe um habitante A que diz a
verdade nos mesmos dias em que A mente, e somente nesses
dias. Em outras palavras, em qualquer dia no qual A minta,
A dir a verdade; em qualquer dia no qual A diga a verdade, A sempre mentir. Uma outra caracterstica dessa ilha
que, para cada par de habitantes A e B, existe um habitante C
que diz a verdade em todos os dias nos quais tanto A como
B dizem a verdade, e em nenhum outro dia (ou seja, C mente

37

em qualquer dia no qual, pelo menos, A ou B tambm minta).


Dizem as ms lnguas que nessa ilha ningum diz a verdade
todos os dias. Esta acusao verdadeira ou no?
5) Uma rainha e seus dois filhos encontram-se prisioneiros no
alto de uma torre. Os pedreiros que construram a torre deixaram uma roldana fixa no topo. Pela roldana passa uma corda
com dois cestos, cada um atado a uma das pontas da corda.
Dentro do cesto que est no cho h uma pedra igual s que
foram utilizadas para construir a torre. A pedra pesa 30 kg. A
rainha descobre que a pedra pode servir de contrapeso nesse
sistema, desde que a diferena de peso entre os dois cestos no
seja superior a 6 kg. Como a rainha pesa 78 kg, a princesa pesa
42 kg e o prncipe, 36 kg, conseguiro eles fugir da torre? O
que aconteceria se, alm da rainha, da princesa, do prncipe e
da pedra, estivessem na torre um porco de 24 kg, um co de 18
kg e um gato de 12 kg, que tambm quisessem fugir da torre,
mas no sassem sozinhos do cesto?
6) Em um certo planeta moravam as galinhas de penacho, que
viviam sempre juntas. Elas, apesar de no se comunicarem entre si, eram inteligentes e tambm observadoras. Mas tinham
dois problemas: s enxergavam o penacho das outras galinhas,
nunca o seu; e, caso qualquer galinha tivesse certeza de que
seu penacho fora cortado, suicidar-se-ia naquela mesma noite.
Uma noite, os gansos do planeta vizinho, sorrateiramente, cortaram o penacho de algumas das galinhas, enquanto elas dormiam. Na manh seguinte, todas as galinhas viram no cho as
penas que indicavam o ataque dos gansos e, na stima noite,
algumas se suicidaram. Quantas galinhas se suicidaram? Por
que isso aconteceu somente na stima noite?
7) Numa viagem, Glria quer alugar um quarto num hotel por
uma semana, mas est sem dinheiro. Ela prope ao gerente pagar cada diria com uma das 7 argolas de ouro de sua pulseira,
mas pretende recuperar as argolas no final da semana, quando sair seu contra-cheque. Para isso, Glria tem que desfazer
a pulseira, e o joalheiro cobra por cada argola que ele abre, e
cobra, mais ainda, para montar novamente a pulseira. Qual
o menor nmero de cortes a serem feitos, de modo que Glria
possa usar como pagamento apenas 1 argola a cada dia?

38

8) Um califa conversa com o calculista:


Tenho cinco lindas escravas. Comprei-as, h poucos meses,
de um prncipe mongol. Dessas cinco encantadoras meninas,
duas tm olhos negros e as outras trs tm os olhos azuis. As
duas escravas de olhos negros, quando interrogadas, dizem
sempre a verdade; as escravas de olhos azuis, ao contrrio, so
mentirosas, isto , nunca dizem a verdade. Dentro de alguns
minutos essas cinco jovens sero conduzidas a este salo e
cada uma delas ter o rosto ocultado por espesso vu (haic),
que a envolver e tornar impossvel distinguir nela o menor
trao fisionmico. Ters que descobrir e indicar, sem a menor
possibilidade de erro, quais so as escravas de olhos negros e
quais so as de olhos azuis. Poders interrogar trs das cinco
escravas, no sendo permitido, em caso algum, fazer mais de
uma pergunta mesma jovem. Com auxlio das trs respostas
obtidas, o problema dever ser solucionado, sendo que a soluo dever ser justificada com todo rigor matemtico.

Califa
Sucessor do profeta Maom,
na qualidade de guia ou lder
temporal e espiritual da comunidade islmica.
Fonte: Dicionrio Houaiss.

O calculista perguntou, ento, a uma das jovens: Qual a cor


de teus olhos? A jovem respondeu em chins, uma lngua
que o calculista no compreendia. Quais foram as outras duas
perguntas, e como ele resolveu o problema?
9) Quantos animais, afinal, eu tenho, se todos so ces, exceto
dois, todos so gatos, exceto dois, e todos so papagaios, exceto dois?
10) Poucas horas viajamos, sem interrupo, pois logo ocorreu
uma curiosa aventura, na qual o homem que calculava ps
em prtica, com grande talento, as suas habilidades de exmio
algebrista. Encontramos perto de um antigo caravanar, quase em abandono, trs homens que discutiam acaloradamente
juntos a uma cfila. O inteligente Ibraim Tavir procurou informar-se do que se tratava:
Somos irmos, disse o mais velho, e recebemos como herana estes 35 camelos. Segundo a vontade de meu pai, devo receber a metade; o meu irmo Hamed Namir, uma tera parte; e,
ao Harim, o mais moo, deve caber apenas a nona parte. No
sabemos, porm, como dividir, desta forma, 35 camelos, pois a

Caravanar
Estalagem pblica, no Oriente Mdio, para hospedar gratuitamente as caravanas que
viajam por regies desrticas; caravanarai, caravancer. Fonte: Dicionrio Houaiss.

39

metade de 35 17,5! Como fazer a partilha, se a tera parte e a


nona parte de 35 tambm no so exatas?
muito simples, replicou o homem que calculava. Encarrego-me de fazer, com justia, essa diviso, se permitirem que eu
junte aos 35 camelos da herana este belo animal que, em boa
hora, aqui nos trouxe!
Como o homem que calculava fez a diviso? Por que a sua
estratgia funcionou?
11) Num sistema solar distante, em torno do sol giram s dois planetas - Oron, habitado pelos oronianos, e Set, habitado pelos
setianos. As duas raas so inteligentes e capazes de viajar de
um planeta para o outro. O problema que, cada vez que os
habitantes de um dos planetas pousam no outro, ficam totalmente desorientados, e tudo aquilo em que acreditam passa a
ser errado! Quando retornam a seu planeta de origem, voltam
a ficar perfeitamente orientados, e tudo em que acreditam fica
outra vez correto. Num certo momento, um habitante de um
desses dois planetas acreditava ser um oroniano e que se encontrava em Set, naquele momento.
Afinal, ele era oroniano ou setiano? Em que planeta ele se
encontrava naquele momento? Em que proposio sempre
acreditaro os habitantes de qualquer dos dois planetas, encontrem-se ou no, no momento, em seu planeta de origem?
Rio das araras ou papagaio
manso, no dialeto Tupi,
nasce em Gois, nas
formaes elevadas
existentes no Parque
Nacional das Emas, reserva
ecolgica, situada na divisa
dos estados de Gois e Mato
Grosso. Pertence bacia
amaznica e marca a divisa
dos estados de Mato Grosso
e Gois, Mato Grosso e
Tocantins e, ainda, Par e
Tocantins, desaguando no
rio Tocantins, na trplice
divisa de Tocantins, Par
e Maranho. Fonte: www.
rioaraguaia.com.br

12) Duas tribos habitam uma ilha no rio Araguaia. A tribo PA s


diz a verdade e a tribo PB sempre mente. Trs nativos, NE, SE e
CO, chegam juntos ilha, numa canoa. Ao chegarem, so obrigados a se identificar, dizendo a que tribo cada um pertence.
Foram feitas as seguintes perguntas, seguidas das respostas
respectivas, a partir das quais voc deve descobrir a que tribo
pertence cada um.
a) Diga-me, NE, SE da aldeia PA?
Sim, ele de PA.
b) Diga-me, SE, NE e CO so da mesma tribo?
No, no so.

40

c) Diga-me, CO, SE de PA?


Sim, ele de PA.
13) Um doblete (doublet) uma seqncia de palavras que comea
por uma palavra dada e termina em outra palavra, tambm
dada, tal que a diferena de duas palavras consecutivas seja
apenas uma letra. Vogais com acentos diferenciais so consideradas como letras iguais, por exemplo, pra e cera. claro
que todas as palavras da seqncia tm a mesma quantidade de letras. Crie dobletes para cada dupla de palavras dadas
abaixo (no valem nomes prprios).
a) sol lua;

e) certo falso;

b) sim no;

f) fogo gua;

c) belo feio;

g) lixo ouro;

d) amor dio;

h) bem mal.

14) Um triplete similar ao doblete, porm, em vez de ter apenas


duas palavras (a inicial e a final), tem trs palavras: a inicial, a
intermediria e a final. Crie tripletes para as triplas abaixo:
a) manh tarde noite;
b) nada meio tudo;
c) pra lima ma.

Soluo dos problemas pares


2) Podemos pensar em resolver o problema de forma algbrica:
A - quantidade de peixes que Natal pescou
P - quantidade de peixes que Pascoal pescou
L - quantidade de peixes que Leonardo, o filho de Natal, pescou
Pf - quantidade de peixes que o filho de Pascoal pescou
Natal pescou o dobro de peixes que pescou seu filho: A = 2L
Pascoal pescou o quntuplo de peixes que pescou seu filho:
P = 5Pf

41

No total pescaram 26 peixes: A + P + L + Pf = 26


Substituindo: 2L + 5Pf + L + Pf = 26, ou 3L + 6Pf = 26
Mas isto impossvel, pois a equao acima uma equao
diofantina que no tem soluo, uma vez que mdc (3, 6) = 3 e
3 no divisor de 26. Temos que rever os graus de parentesco!
Como Leonardo filho de Natal, ento Natal deve ser filho de
Pascoal. Vamos verificar:
A = 2L ; P = 5A = 10L ; A + P + L = 26
Substituindo: 2L + 10L + L = 26 L = 2
Leonardo pescou 2 peixes, Natal pescou 4 peixes e Pascoal pescou 20 peixes. Natal o nome do filho de Pascoal e ele pescou
4 peixes.
4) Vamos partir de um habitante A. Para ele, existe um habitante
A cujos hbitos em relao verdade e mentira so opostos
aos seus. Usando a segunda caracterstica, com A fazendo o
papel de B, existe um habitante C que diz a verdade somente
nos dias em que tanto A como A dizem a verdade. Como A e
A nunca dizem a verdade no mesmo dia, ento C mente todo
dia. Mas, para este habitante C, existe um habitante C cujo
comportamento sempre oposto ao de C e, portanto, C diz a
verdade todo dia. Assim, a acusao no verdadeira.
6) Qualquer galinha que, ao amanhecer seguinte noite do ataque, visse todas as demais com penacho, concluiria que somente ela estava sem penacho e, assim, deveria se suicidar na
primeira noite. Como no houve suicdio na primeira noite,
ao amanhecer do segundo dia as inteligentes galinhas concluram: as vtimas foram duas ou mais! Se alguma estivesse
vendo todas as outras com penacho e apenas uma delas sem,
concluiria que ela mesma estava sem penacho. Logo, ambas
se suicidariam na segunda noite. Ao clarear do terceiro dia,
no entanto, estavam todas l! Ento o nmero de vtimas seria maior ou igual a 3. Qualquer galinha que visse duas sem
penacho concluiria que ela era a terceira vtima e, assim, as
trs se suicidariam na terceira noite. A ausncia de suicdios na
terceira noite fez com que no amanhecer do quarto dia todas

42

conclussem que o nmero de vtimas era maior ou igual a 4.


Continuando com este raciocnio, no stimo dia, a galinha que
visse seis outras sem penacho teria certeza que era a stima vtima e todas as sete galinhas se suicidariam na stima noite.
8) As outras duas perguntas e respectivas respostas foram as seguintes. O que sua companheira acabou de falar? A resposta
foi: Os meus olhos so azuis. A prxima pergunta foi: Qual
a cor dos olhos dessas duas jovens que acabo de interrogar? A
resposta foi, ento: A primeira tem olhos negros e a segunda
tem olhos azuis.
Faa a redao do argumento do calculista, identificando as
jovens de olhos azuis e as de olhos negros.
10) Ao juntar mais um camelo aos 35, o total passou a ser 36, que
divisvel por 2, por 3 e por 9. Todos os envolvidos receberiam
mais fraes de camelo, pois 362 = 18, 363 = 12 e 369 = 4,
ficando satisfeitos. Como 18 + 12 + 4 = 34, sobraram 2 camelos
no final.
12) Se NE falou a verdade, ento NE de PA, assim como SE. Pela
segunda resposta, CO seria de PB. Mas, a terceira resposta
inconsistente com esses dados, pois CO falaria a verdade e, assim, pertenceria tribo PA. Sendo assim, NE mentiu. Ou seja,
NE e SE so de PB. Logo, CO de PB, pela segunda resposta,
que consistente com a resposta que CO d, que falsa.
14) a) manh manha manta marta farta farda
tarda tarde tardo tordo mordo morto morte
norte noite.
b) nada nata neta meta meia meio medo
mudo tudo.
c) pra cera cela tela tema lema lima lama
cama caa maa ma.

Marta
um mamfero, parecido
com a doninha e comum no
hemisfrio norte.
Maa
Clava.

43

Referncias
[1] CARROLL, Lewis. Alice no Pas das Maravilhas; Atravs do Espelho. Rio de Janeiro: Jorge Zahar, 2002.
[2] ___________. Aventuras de Alice no Pas das Maravilhas Atravs
do espelho e o que Alice encontrou l: Outros textos. So Paulo: Summus, 1980.
[3] GARDNER, Martin. Ah, Descobri! Lisboa: Gradiva, 1990.
[4] ___________. Ah, Apanhei-te! Lisboa: Gradiva, 1993.
[5] MALBA TAHAN. O homem que calculava. 56. ed. Rio de Janeiro:
Record, 2002.

Captulo 3
Problemas Olmpicos

47

Captulo 3
Problemas Olmpicos
3.1 Introduo
Neste captulo, apresentamos um conjunto de problemas selecionados entre o conjunto daqueles utilizados para treinamento
ou aplicados em alguma fase das olimpadas de matemtica. A
maior parte vem dos arquivos da Olimpada Brasileira de Matemtica (OBM), e so organizados em trs nveis: nvel 1, quinta e
sexta sries; nvel 2, stima e oitava sries e nvel 3, as trs sries
do Ensino Mdio.
O programa de olimpadas de matemtica existe no Brasil h
mais de 20 anos. Desde 1998, a Sociedade Brasileira de Matemtica tem estimulado a realizao de Olimpadas Regionais, com
o objetivo de contribuir para a melhoria do ensino de matemtica, incentivando alunos e professores a resolver problemas que
no so apresentados usualmente em livros didticos. Problemas
olmpicos necessitam de imaginao, criatividade e conhecimento real do contedo. Eles desafiam professores, alunos e amantes
da matemtica em geral.

Olimpada Brasileira de
Matemtica das Escolas
Pblicas http://www.
obmep.org.br

Nossa experincia de vrios anos em um projeto de extenso da


UFSC, chamado Olimpada Regional de Matemtica, levou-nos a
trabalhar com problemas olmpicos em diversos ambientes: alunos da rede pblica de ensino (OBMEP), cursos de formao continuada para professores etc., com os seguintes objetivos:
i) discutir os contedos bsicos sob um ponto de vista que no
aparece, em geral, nos livros didticos;
ii) introduzir professores e estudantes em atividades de olimpada de matemtica, incentivando-os a abordar problemas
olmpicos como desafios intelectuais;
iii) incentivar professores e estudantes busca de novas abordagens de contedos bsicos, via problemas olmpicos.

48

Observamos que a realizao de olimpadas de matemtica nas escolas tem um aspecto dinamizador, integrador e propagador, mobilizando alunos, professores e pais na resoluo de problemas. H
escolas que, inclusive, organizam olimpadas nas quais pais e filhos
resolvem juntos os problemas propostos.
Apresentamos, primeiramente, as provas da primeira fase da XXVI
Olimpada Brasileira de Matemtica, nos 3 nveis oficiais, realizadas
em 5 de junho de 2004. Observe que h problemas comuns a dois
nveis.
Nosso objetivo, aqui, introduzir essas provas de olimpada do
modo como foram apresentadas aos alunos de vrios estados do
pas que nelas se inscreveram. Os gabaritos encontram-se no stio
da OBM. No entanto, importante que cada um procure suas prprias solues. Na seqncia, propomos um conjunto de problemas
olmpicos (ou seja, que foram aplicados em olimpadas de matemtica) dos vrios nveis oficiais.
Pensar sobre um problema e discuti-lo com os colegas mais importante do que v-lo pronto. Divirta-se!

3.2 XXVI OBM Nvel 1 1 fase


1) Calcule o valor de 1.997 + 2.004 + 2.996 + 4.003.
a) 10.000

b) 11.000

c) 10.900

d) 12.000

e) 13.000

2) Qual dos nmeros a seguir mpar?


a) 7 x 8

b) 37 23

c) 9 x 36

d) 144 : 36

3) Quanto 26 + 26 + 26 + 26 44?
a) 0

b) 2

c) 4

d) 42 e) 44

4) 20% de 40 igual a
a) 5

b) 8

c) 10 d) 12 e) 20

e) 17 x 61

http://www.obm.org.br

49

5) Simplificando a frao

a) 2004

b)

113
355

c)

2004 + 2004
, obtemos:
2004 + 2004 + 2004

1
2004

d)

2
3

e)

2
7

6) Os alunos de uma escola participaram de uma excurso para


a qual dois nibus foram contratados. Quando os nibus chegaram, 57 alunos entraram no primeiro nibus e apenas 31 no
segundo. Quantos alunos devem passar do primeiro para o
segundo nibus para que a mesma quantidade de alunos seja
transportada nos dois nibus?
a) 8

b) 13 c) 16 d) 26 e) 31

7) Uma professora tem 237 balas para dar a seus 31 alunos. Qual
o nmero mnimo de balas a mais que ela precisa conseguir
para que todos os alunos recebam a mesma quantidade de balas, sem sobrar nenhuma para ela?
a) 11

b) 20 c) 21 d) 31 e) 41

8) Dezoito quadrados iguais so construdos e sombreados como


mostra a figura. Qual frao da rea total sombreada?

Figura 3.1

a)

7
b)
18

c)

d)

e)

9) O preo de uma corrida de txi igual a R$2,50 (bandeirada),


mais R$0,10 por cada 100 metros rodados. Tenho apenas R$10,00
no bolso. Logo, tenho dinheiro para uma corrida de at:
a) 2,5 km

b) 5,0 km

c) 7,5 km

d) 10,0 km

e) 12,5 km

50

10) Um arquiteto apresenta ao seu cliente cinco plantas diferentes para o projeto de ajardinamento de um terreno retangular,
onde as linhas cheias representam a cerca que deve ser construda para proteger as flores. As regies claras so todas retangulares e o tipo de cerca o mesmo em todos os casos. Em
qual dos projetos o custo da construo da cerca ser maior?

a)

b)

c)

d)

e)

Figura 3.2

11) 108 crianas da 5 e 6 sries vo fazer um passeio a uma caverna. So formados grupos iguais com mais de 5, porm, menos
de 20 alunos. Com relao ao nmero de estudantes por grupo, de quantas formas diferentes eles podem ser feitos?
a) 2

b) 8 c) 5

d) 4

e) 3

12) O desenho ao lado (fig. 3.3) mostra o mapa de um pas (imaginrio) constitudo por cinco estados. Deseja-se colorir este
mapa com as cores verde, azul e amarela, de modo que dois
estados vizinhos no possuam a mesma cor. De quantas maneiras diferentes o mapa pode ser pintado?
a) 12

b) 6

c) 10 d) 24 e) 120

13) Um arteso comea a trabalhar s 8h e produz 6 braceletes a


cada vinte minutos; seu auxiliar comea a trabalhar uma hora
depois e produz 8 braceletes do mesmo tipo a cada meia hora.
O arteso pra de trabalhar s 12h, mas avisa ao seu auxiliar
que este dever continuar trabalhando at produzir o mesmo
que ele. A que horas o auxiliar ir parar?
a) 12h b) 12h30min c) 13h d) 13h30min e) 14h30min
14) O algarismo das unidades do nmero 1 3 5 97 99
a) 1

b) 3

c) 5

d) 7

e) 9

Figura 3.3

51

15) Dois quadrados, cada um com rea 25 cm2, so colocados lado


a lado para formar um retngulo. Qual o permetro do retngulo?
a) 30 cm b) 25 cm c) 50 cm

d) 20 cm e) 15 cm

16) Se girarmos o pentgono regular, abaixo, em um ngulo de


252 em torno do seu centro, no sentido horrio, que figura
ser obtida?

Figura 3.4

a)

b)

c)

d)

e)

Figura 3.5

17) Os resultados de uma pesquisa das cores de cabelo de 1.200


pessoas so mostrados no grfico abaixo.

castanho
30%

preto
24%
ruivo
16%

loiro
?

Figura 3.6

Quantas dessas pessoas possuem o cabelo loiro?


a) 60

b) 320

c) 360

d) 400

e) 840

52

18) Um cubo pode ser construdo a partir dos dois pedaos de


papelo apresentados em uma das alternativas a seguir, bastando apenas dobrar nas linhas tracejadas e unir nas linhas
contnuas. Esses dois pedaos so:
a)

b)

d)

e)

c)

Figura 3.7

19) Ao somar cinco nmeros consecutivos em sua calculadora,


Esmeralda encontrou um nmero de 4 algarismos: 2 0 0 *. O
ltimo algarismo no est ntido, pois o visor da calculadora
est arranhado, mas ela sabe que ele no zero. Este algarismo
s pode ser:
a) 5

b) 4

c) 3

d) 2

e) 9

20) Sobre uma mesa esto trs caixas e trs objetos. Cada objeto
est em uma caixa diferente: uma moeda, um grampo e uma
borracha. Sabe-se que
a caixa verde est esquerda da caixa azul;
a moeda est esquerda da borracha;
a caixa vermelha est direita do grampo;
a borracha est direita da caixa vermelha.
Em que caixa est a moeda?
a) Na caixa vermelha
b) Na caixa verde
c) Na caixa azul

53

d) As informaes fornecidas so insuficientes para se dar


uma resposta
e) As informaes fornecidas so contraditrias.
21) Um feirante vende batatas e, para pes-las, utiliza uma balana de dois pratos, um peso de 1 kg, um peso de 3 kg e um
peso de 10 kg. Considere a seguinte afirmao: Este feirante
consegue pesar (com uma nica pesagem) n quilogramas de
batatas. Quantos valores positivos de n tornam essa afirmao verdadeira, supondo que ele pode colocar pesos nos dois
pratos?
a) 7

b) 10

c) 12

d) 13

e) 14

22) O mapa abaixo mostra um conjunto residencial onde as casas, numeradas, so interligadas por 23 ruelas. O vendedor Z
Ruela, que mora na casa 8, planeja passar por todas as outras
casas e retornar sua, percorrendo o menor nmero possvel
de ruelas. Ele deixar de caminhar por quantas ruelas?
2

1
5

4
8

9
10

11

12

Figura 3.8

a) 15

b) 10

c) 13 d) 12 e) 11

23) O arranjo a seguir, composto por 32 hexgonos, foi montado


com varetas, todas com comprimento igual ao lado do hexgono. Quantas varetas, no mnimo, so necessrias para montar
o arranjo?
a) 113
Figura 3.9

b) 123

c) 122

d) 132

e) 152

54

24) Observe a figura:

Figura 3.10

Duas das figuras abaixo representam o objeto acima colocado


em outras posies.
I)

II)

III)

IV)

Figura 3.11

Elas so:
a) I e II

b) I e IV

c) II e IV

d) I e III

e) II e III

25) Entre 1986 e 1989, poca em que voc ainda no tinha nascido,
a moeda do pas era o cruzado (Cz$). Com a imensa inflao
que tivemos, a moeda foi mudada algumas vezes: tivemos o
cruzado novo, o cruzeiro, o cruzeiro real e, finalmente, o real.
A converso entre o cruzado e o real : 1 real = 2.750.000.000
cruzados. Imagine que a moeda no tivesse mudado e que
Joo, que ganha hoje 640 reais por ms, tivesse que receber seu
salrio em notas novas de 1 cruzado. Se uma pilha de 100 notas novas tem 1,5 cm de altura, o salrio em cruzados de Joo
faria uma pilha de altura:
a) 26,4 km

b) 264 km

d) 264.000 km

c) 26.400 km

e) 2.640.000 km

O Plano Cruzado foi


um plano econmico,
lanado em 1 de maro de
1986, por Dilson Funaro,
ministro da Fazenda do
governo de Sarney. O
plano mudou a moeda
do Brasil de cruzeiro para
cruzado novo, congelou
os preos e salrios e
criou o gatilho salarial e o
seguro-desemprego. Fonte:
Wikipdia, a enciclopdia
livre.

55

3.3 XXVI OBM Nvel 2 1a fase


1) Quanto 26 + 26 + 26 + 26 44?
a) 0

b) 2

c) 4

d) 42 e) 44

2) Se m e n so inteiros no negativos com m < n, definimos m n


como a soma dos inteiros entre m e n, incluindo m e n. Por
exemplo, 5 8 = 5 + 6 + 7 + 8 = 26.
O valor numrico de

22 26
:
46

a) 4

d) 10 e) 12

b) 6

c) 8

3) Entre 1986 e 1989, poca em que vocs ainda no tinham nascido, a moeda do pas era o cruzado (Cz$). Com a imensa inflao que tivemos, a moeda foi mudada algumas vezes: tivemos
o cruzado novo, o cruzeiro, o cruzeiro real e, finalmente, o real.
A converso entre o cruzado e o real : 1 real = 2.750.000.000
cruzados. Imagine que a moeda no tivesse mudado e que
Joo, que ganha hoje 640 reais por ms, tivesse que receber seu
salrio em notas novas de 1 cruzado. Se uma pilha de 100 notas novas tem 1,5 cm de altura, o salrio em cruzados de Joo
faria uma pilha de altura:
a) 26,4km

b) 264km

d) 264.000km

c) 26.400km

e) 2.640.000km

4) O arranjo a seguir, composto por 32 hexgonos, foi montado


com varetas, todas com comprimento igual ao lado do hexgono. Quantas varetas, no mnimo, so necessrias para montar
o arranjo?

Figura 3.12

a) 113 b) 123 c) 122 d) 132 e) 152

56

5) O algarismo das unidades do nmero 1 3 5 97 99 :


a) 1

b) 3

c) 5

d) 7

e) 9

6) Se girarmos o pentgono regular, abaixo, em um ngulo de


252 em torno do seu centro, no sentido horrio, que figura
ser obtida?

Figura 3.13

a)

b)

c)

d)

e)

Figura 3.14

7) H 1.002 balas de banana e 1.002 balas de ma numa caixa.


Lara tira, sem olhar o sabor, duas balas da caixa. Seja p a probabilidade de as duas balas serem do mesmo sabor e seja q a
probabilidade de as duas balas serem de sabores diferentes.
Quanto vale a diferena entre p e q?
a) 0

b)

1

2004

c)

d)

e)

8) O permetro de um retngulo 100 e a diagonal mede x. Qual


a rea do retngulo?
x2
x2
a) 625 x2

b) 625

c) 1.250
2
2
d) 250

x2

2

e) 2.500

x2
2

9) Ao somar cinco nmeros consecutivos em sua calculadora,


Esmeralda encontrou um nmero de 4 algarismos: 2 0 0 *. O
ltimo algarismo no est ntido, pois o visor da calculadora

57

est arranhado, mas ela sabe que ele no zero. Este algarismo
s pode ser:
a) 5

b) 4

c) 3

d) 2

e) 9

2004
10) Para quantos inteiros positivos m o nmero 2
um inteim 2
ro positivo?
a) um

b) dois

c) trs

d) quatro

e) mais do que quatro

11) Se x + y = 8 e xy = 15, qual o valor de x2 + 6xy + y2?


a) 64

b) 109

c) 120

d) 124

e) 154

12) Dois espelhos formam um ngulo de 30 no ponto V. Um raio


de luz, vindo de uma fonte S, emitido paralelamente a um
dos espelhos e refletido pelo outro espelho no ponto A, como
mostra a figura. Depois de uma certa quantidade de reflexes,
o raio retorna a S. Se AS e AV tm 1 metro de comprimento, a
distncia percorrida pelo raio de luz, em metros,
A

30

Figura 3.15

a) 2

b) 2 + 3

c) 1 + 2 + 3

d)

13) Na figura, quanto vale x?


a) 6

b) 12 c) 18 d) 20

Figura 3.16

e) 24

2(1 + 3)

e) 5 3

58
2x
x
14) Se 2 (2 ) = 4 + 64 , ento x igual a:

a) 2

b) 1

c) 1

d) 2

e) 3

15) Qual o maior valor da soma dos algarismos de um nmero


de trs algarismos?
a) 7

b) 8

c) 9

d) 10

e) 11

16) Um arquiteto apresenta ao seu cliente cinco plantas diferentes para o projeto de ajardinamento de um terreno retangular,
onde as linhas cheias representam a cerca que deve ser construda para proteger as flores. As regies claras so todas retangulares e o tipo de cerca o mesmo em todos os casos. Em
qual dos projetos o custo de construo da cerca ser maior?

a)

b)

c)

d)

e)

Figura 3.17

17) Um ponto P pertence ao interior de um quadrado com 10 cm


de lado. No mximo, quantos pontos da borda do quadrado
podem estar a uma distncia de 6 cm do ponto P?
a) 1

b) 2

c) 4

d) 6

e) 8

18) Um cubo pode ser construdo, a partir dos dois pedaos de


papelo apresentados em uma das alternativas a seguir, bastando apenas dobrar nas linhas tracejadas e unir nas linhas
contnuas. Esses dois pedaos so:
a)

b)

d)

e)

Figura 3.18

c)

59

19) No tringulo PQR, a altura PF divide o lado QR em dois segmentos de medidas


QF = 9 e RF = 5. Se PR = 13, qual a medida de PQ?
a) 5

b) 10

c) 15

d) 20

e) 25

20) Sobre uma mesa esto trs caixas e trs objetos. Cada objeto
est em uma caixa diferente: uma moeda, um grampo e uma
borracha. Sabe-se que:
a caixa verde est esquerda da caixa azul;
a moeda est esquerda da borracha;
a caixa vermelha est direita do grampo;
a borracha est direita da caixa vermelha.
Em que caixa est a moeda?
a) Na caixa vermelha
b) Na caixa verde
c) Na caixa azul
d) As informaes fornecidas so insuficientes para se dar
uma resposta
e) As informaes fornecidas so contraditrias.
21) No desenho abaixo, o quadriltero ABCD um quadrado de
lado 3 cm e os tringulos ABF e AED so ambos equilteros.
Qual a rea da regio destacada?
A

B
F

C
Figura 3.19

a) 2 cm2

b) 1,5 cm2

c) 3 cm2

d) 4,5 cm2

e) 2,5 cm2

60

22) Uma folha quadrada foi cortada em 42 quadrados menores,


dos quais um tem rea maior do que 1 cm2 e os demais tm
rea de 1 cm2. Qual a medida do lado da folha?
a) 6 cm

b) 12 cm

c) 21 cm

d) 19 cm

e) 20 cm

23) Eu planejava fazer um curral quadrado, com uma certa rea,


usando uma certa quantidade de cerca de arame farpado. Descobri, porm, que tenho 10% a menos de cerca do que esperava. Por esta razo, a rea cercada ser:
a) 5% menor

b) 10% menor

d) 20% menor

c) 19% menor

e) 25% menor

24) Um arteso comea a trabalhar s 8h e produz 6 braceletes a


cada vinte minutos; seu auxiliar comea a trabalhar uma hora
depois e produz 8 braceletes do mesmo tipo a cada meia hora.
O arteso pra de trabalhar s 12h mas avisa ao seu auxiliar
que este dever continuar trabalhando at produzir o mesmo
que ele. A que horas o auxiliar ir parar?
a) 12h

b) 12h30min

c) 13h

d) 13h30min

e) 14h30min

25) Esmeralda, a digitadora, tentou digitar um nmero de seis algarismos, mas os dois algarismos 1 no apareceram (a tecla
devia estar com defeito). O que apareceu foi 2.004. Quantos
so os nmeros de seis algarismos que ela pode ter tentado
digitar?
a) 4

b) 8

c) 10

d) 15

e) 20

3.4 XXVI OBM Nvel 3 1 fase


1) A funo f dada pela tabela a seguir.
x

f (x)

Por exemplo, f (2) = 1. Quanto vale f ( f ( ( f (4)))) , 2.004 vezes?


a) 1

b) 2

c) 3

d) 4

e) 5

61

2) Seja AB um segmento de comprimento 26, e sejam C e D pontos sobre o segmento AB tais que AC = 1 e AD = 8. Sejam E e F
pontos sobre uma semicircunferncia de dimetro AB, sendo
EC e FD perpendiculares a AB. Quanto mede o segmento EF?
a) 5

b)

c) 7

d)

e) 12

3) As alturas de um tringulo medem 12, 15 e 20. O maior ngulo


interno do tringulo mede
a) 72

b) 75

c) 90

d) 108

e) 120

4) Esmeralda, a digitadora, tentou digitar um nmero de seis algarismos, mas os dois algarismos 1 no apareceram (a tecla devia estar com defeito). O que apareceu foi 2.004. Quantos so os
nmeros de seis algarismos que ela pode ter tentado digitar?
a) 4

b) 8

c) 10

d) 15

e) 20

5) O produto dos nmeros que aparecem nas alternativas incorretas desta questo um cubo perfeito.
Assinale a alternativa correta.
a) 4

b) 8

c) 18

d) 54

e) 192

6) Qual o menor inteiro positivo n para o qual qualquer subconjunto de n elementos de


contm dois nmeros cuja
diferena 8?
a) 2

b) 8

c) 12

d) 13

7) Sejam
a=

12 22 32
10012
+
+ + +
1 3
5
2001
e
.

Qual o inteiro mais prximo de a b?


a) 500

b) 501

c) 999

d) 1.000

e) 1.001

e) 15

62

8) Uma ampulheta formada por dois cones idnticos. Inicialmente, o cone superior est cheio de areia e o cone inferior est
vazio. A areia flui do cone superior para o inferior com vazo
constante. O cone superior se esvazia em exatamente uma hora
e meia. Quanto tempo demora at que a altura da areia no cone
inferior seja metade da altura da areia no cone superior?
a) 30min

b) 10h

d) 1h10min12s

c) 1h03min20s

e) 1h14min30s

9) A funo real f, definida no conjunto dos inteiros, satisfaz


f (n) (n + 1) f (2 n) = (n + 3) 2 , para todo n inteiro. Quanto vale
f (0) ?
a) 17

b) 0

c) 1

d) 2

e) 9

10) Com trs algarismos distintos a, b e c, possvel formar 6 nmeros de dois algarismos distintos. Quantos conjuntos {a , b , c}
so tais que a soma dos 6 nmeros formados 484?
a) Um

b) Dois

c) Trs

d) Quatro

e) Mais que quatro

11) Dois cubos tm faces pintadas de ocre ou magenta. O primeiro


cubo tem cinco faces ocres e uma face magenta. Quando os dois
cubos so lanados, a probabilidade de as faces viradas para
cima dos dois cubos serem da mesma cor (sim, ocre e magenta
so cores!) 1/2. Quantas faces ocres tem o segundo cubo?
a) 1

b) 2

c) 3

d) 4

e) 5

12) Para quantos inteiros positivos m o nmero 2004 um inteim2 2


ro positivo?
a) Um

b) Dois

c) Trs

d) Quatro

e) Mais que quatro

13) Dois espelhos formam um ngulo de 30o no ponto V. Um raio


de luz, vindo de uma fonte S, emitido paralelamente a um
dos espelhos e refletido pelo outro espelho no ponto A, como
mostra a figura 3.20. Depois de uma certa quantidade de reflexes, o raio retorna a S. Se AS e AV tm 1 metro de comprimento, a distncia percorrida pelo raio de luz, em metros,

63

30

Figura 3.20

a) 2

b) 2 + 3

c) 1 + 2 + 3

d)

2(1 + 3)

e) 5 3

14) Para n inteiro positivo, definimos n! (l-se n fatorial) o produto de todos os inteiros positivos menores ou iguais a n. Por
exemplo, 6! = 1 2 3 4 5 6.
Se n! = 215 36 53 72 11 13, ento n igual a
a) 13

b) 14

c) 15

d) 16

e) 17

15) Constri-se o quadrado ABXY sobre o lado AB do heptgono


regular ABCDEFG, exteriormente ao heptgono. Determine a
, em radianos.
medida do ngulo BXC
a)

b)

3

7

c)


14

d)

e)

3
28

16) O conjunto das razes reais da equao


x + 2 x 1 + x 2 x 1 = 2

a) {1}
D

2
B

b) {1,2}

c) [1, 2]

d) ]1, 2[

e) {2}

17) No desenho ao lado, os segmentos AB e CD so perpendiculares ao segmento BC. Sabendo que o ponto
M pertence ao segmento AD e que o tringulo BMC
retngulo no issceles, qual a rea do tringulo
ABM ?
a) 1

b)

c)

d)

e)

9
5

Figura 3.21

18) Entre 1986 e 1989, a moeda do pas era o cruzado (Cz$). Com
a imensa inflao que tivemos, a moeda foi mudada algumas
vezes: tivemos o cruzado novo, o cruzeiro, o cruzeiro real e,
finalmente, o real. A converso entre o cruzado e o real : 1

64

real = 2.750.000.000 cruzados. Imagine que a moeda no tivesse mudado e que Joo, que ganha hoje 640 reais por ms, tivesse que receber seu salrio em notas novas de 1 cruzado. Se
uma pilha de 100 notas novas tem 1,5 cm de altura, o salrio
em cruzados de Joo faria uma pilha de altura:
a) 26,4 km

b) 264 km

c) 2.640 km

d) 26.400 km

e) 264.000 km

19) O dono de uma loja empilhou vrios blocos medindo 0,8m x


0,8m x 0,8m no canto da loja e encostados numa parede de
vidro que d para a rua, conforme mostra a figura ao lado.
Quantos blocos, no mximo, uma pessoa de 1,80m de altura
que est do lado de fora da loja pode enxergar?
Obs. Consideramos que uma pessoa pode enxergar uma caixa
se consegue ver uma pequena regio de rea positiva de sua
superfcie.
a) 13

b) 14

c) 15

d) 16

Figura 3.22

e) 17

20) O arranjo a seguir, composto por 32 hexgonos, foi montado


com varetas, todas com comprimento igual ao lado do hexgono. Quantas varetas, no mnimo, so necessrias para montar
o arranjo?
a) 113

b) 123

c) 122

d) 132

e) 152

21) Numa prova para uma sala com 30 alunos, a mdia aritmtica
das 10 piores notas 3 e a mdia aritmtica das 10 melhores
notas 9. O menor valor possvel e o maior valor possvel para
a mdia da sala so, respectivamente:
a) 6 e 7

b) 5 e 7

c) 4 e 6

d) 3 e 9

e) 4 e 8

22) Sobre uma mesa esto trs caixas e trs objetos. Cada objeto
est em uma caixa diferente: uma moeda, um grampo e uma
borracha. Sabe-se que
a caixa verde est esquerda da caixa azul;
a moeda est esquerda da borracha;

Figura 3.23

65

a caixa vermelha est direita do grampo;


a borracha est direita da caixa vermelha.
Em que caixa est a moeda?
a) Na caixa vermelha
b) Na caixa verde
c) Na caixa azul
d) As informaes fornecidas so insuficientes para se dar
uma resposta
e) As informaes fornecidas so contraditrias.
23) Esmeralda, a digitadora, queria digitar um nmero N de dois
algarismos que quadrado perfeito, mas se enganou, trocando cada algarismo pelo seu sucessor (afinal, as teclas so vizinhas!). Por uma grande coincidncia, o nmero digitado tambm quadrado perfeito! Qual a soma dos algarismos de N?
a) 4

b) 5

c) 6

d) 7

e) 8

24) Esmeralda escreveu (corretamente!) todos os nmeros de 1 a


999, um atrs do outro: 12345678910111213 997998999.
Quantas vezes aparece o agrupamento 21, nesta ordem?
a) 11

b) 21

c) 31

d) 41

e) 51

25) Um feirante vende batatas e, para pes-las, utiliza uma balana


de dois pratos, um peso de 1 kg, um peso de 3 kg e um peso de
10 kg. Considere a seguinte afirmao: Este feirante consegue
pesar (com uma nica pesagem) n quilogramas de batatas.
Quantos valores positivos de n tornam essa afirmao verdadeira, supondo que ele pode colocar pesos nos dois pratos?
a) 7

b) 10

c) 12

d) 13

e) 14

66

3.5 Problemas olmpicos - miscelnea


1) Num relgio digital, que marca de 0:00 at 23:59, quantas vezes
por dia o mostrador apresenta todos os algarismos iguais?
2) A prefeitura de uma certa cidade fez uma campanha que permite trocar 4 garrafas de 1 litro vazias por uma garrafa de 1
litro cheia de leite. At quantos litros de leite pode obter uma
pessoa que possua 43 dessas garrafas vazias?
3) O nmero 10 pode ser escrito de duas formas como soma de
dois nmeros primos: 10 = 5 + 5 e 10 = 3 + 7. De quantas maneiras podemos expressar o nmero 25 como uma soma de dois
nmeros primos?
4) 1 litro de lcool custa R$0,75. O carro de Henrique percorre 25
km com 3 litros de lcool. Quantos reais sero gastos em lcool
para percorrer 600 km?
5) H 18 anos Hlio tinha precisamente trs vezes a idade de seu
filho. Agora tem o dobro da idade desse filho. Quantos anos
tm Hlio e seu filho?
6) Se os nmeros naturais so colocados em colunas, como se mostra abaixo, debaixo de que letra aparecer o nmero 2.000?
A

C
2

9
10

11

12

I
5

6
13

16
21

4
7

17
20

3
8

18
19

14
15

...

...

7) Os 61 aprovados em um concurso, cujas notas foram todas distintas, foram distribudos em duas turmas, de acordo com a
nota obtida: os 31 primeiros foram colocados na turma A e os
30 seguintes na turma B. As mdias das duas turmas no concurso foram calculadas. Depois, no entanto, decidiu-se passar
o ltimo colocado da turma A para a turma B. Com isso:

67

a) A mdia da turma A melhorou, mas a da B piorou.


b) A mdia da turma A piorou, mas a da B melhorou.
c) As mdias de ambas as turmas melhoraram.
d) As mdias de ambas as turmas pioraram.
e) As mdias das turmas podem melhorar ou piorar, dependendo das notas dos candidatos.
8) Numa caixa havia vrias bolas, sendo 5 azuis, 4 amarelas, 3
vermelhas, 2 brancas e 1 preta. Renato retirou 3 bolas da caixa.
Sabendo que nenhuma delas era azul, amarela, ou preta, podemos afirmar a respeito dessas 3 bolas que:
a) so da mesma cor
b) uma vermelha e duas so brancas
c) uma branca e duas so vermelhas
d) pelo menos uma vermelha
9) Quatro amigos vo visitar um museu e um deles resolve entrar sem pagar. Aparece um fiscal que quer saber qual deles
entrou sem pagar.
Eu no fui, diz o Benjamim.
Foi o Carlos, diz o Mrio.
Foi o Pedro, diz o Carlos.
O Mrio no tem razo, diz o Pedro.
S um deles mentiu. Quem no pagou a entrada do museu?
10) Escrevem-se, em ordem crescente, os nmeros inteiros e positivos que sejam mltiplos de 7 ou de 8 (ou de ambos), obtendose 7, 8, 14, 16, ... Qual o 100 nmero escrito?
11) Numa certa cidade o metr tem todas suas 12 estaes em linha
reta. A distncia entre duas estaes vizinhas sempre a mesma. Sabe-se que a distncia entre a terceira e a sexta estaes
igual a 3.300 metros. Qual o comprimento dessa linha?

68

12) Quantos nmeros de dois algarismos so primos e tm como


antecessor um quadrado perfeito?
13) Quantas vezes num dia (24 horas) os ponteiros de um relgio
formam ngulo reto?
14) Uma fbrica embala 8 latas de palmito em caixas de papelo
cbicas de 20 cm de lado. Para que possam ser mais bem transportadas, essas caixas so colocadas, da melhor maneira possvel, em caixotes de madeira de 80 cm de largura por 120 cm de
comprimento por 60 cm de altura. Qual o nmero de latas de
palmito em cada caixote?
15) Qual dos nmeros maior:
a) 345

b) 920

c) 2714

d) 2439

e) 8112

16) Renata digitou um nmero em sua calculadora, multiplicou-o


por 3, somou 12, dividiu o resultado por 7 e obteve 15. Qual
o nmero digitado por Renata?
17) Numa competio de ciclismo, Carlinhos d uma volta completa na pista em 30 segundos, enquanto Paulinho leva 32 segundos para completar uma volta. Quando Carlinhos completar a
volta de nmero 80, que volta estar completando Paulinho?
18) Quantos nmeros de trs algarismos existem cuja soma dos
algarismos 25?
19) Um estacionamento para carros cobra 1 real pela primeira hora
e 75 centavos a cada hora ou frao de hora seguinte. Andr estacionou seu carro s 11h20min e saiu s 15h40min. Quantos
reais ele deve pagar pelo estacionamento?
20) A soma de todos os nmeros mpares de dois algarismos menos a soma de todos os nmeros pares de dois algarismos ?
21) Joo mais velho que Pedro, que mais novo que Carlos; Antonio mais velho do que Carlos, que mais novo do que Joo.
Antnio no mais novo do que Joo e todos os quatro meninos tm idades diferentes. Qual o mais jovem?

69

22) Uma fazenda retangular possui 10 km de largura por 20 km


de comprimento e foi desapropriada para reforma agrria. Se
a fazenda deve ser dividida para 200 famlias de modo que
todas as famlias recebam a mesma rea, quantos metros quadrados receber cada famlia?
23) Para fazer 12 bolinhos, preciso de, exatamente, 100 g de acar,
50 g de manteiga, meio litro de leite e 400 g de farinha. Qual
a maior quantidade de bolinhos que posso fazer com 500 g de
acar, 300 g de manteiga, 4 litros de leite e 5 kg de farinha?
24) No planeta Z, todos os habitantes possuem 3 pernas e cada carro possui 5 rodas. Em uma pequena cidade desse planeta, existem ao todo 97 pernas e rodas. Ento, podemos afirmar que:
a) possvel que existam 19 carros nessa cidade
b) existem, no mximo, 17 carros nessa cidade
c) essa cidade tem 9 habitantes e 14 carros
d) essa cidade possui, no mximo, 16 carros
e) nessa cidade existem mais carros que pessoas
25) Ronaldo, sempre que pode, guarda moedas de 50 centavos ou
1 real. Atualmente ele tem 100 moedas, num total de 76 reais.
Quantas moedas de um valor ele tem a mais do que a de outro
valor?
26) Seis cartes com nmeros somente numa das faces so colocados sobre uma mesa, conforme a ilustrao. Os cartes X e Y
esto com a face numerada voltada para baixo. A mdia aritmtica dos nmeros de todos os cartes 5. A mdia aritmtica dos nmeros do carto Y e de seus trs vizinhos 3. Qual
o nmero escrito no carto X?
8

27) Considere dois nmeros naturais, cada um deles com trs algarismos diferentes. O maior deles s tem algarismos pares e o

70

menor deles s tem algarismos mpares. Calcule o menor valor


possvel da diferena entre eles.
28) Joana escreve a seqncia de nmeros naturais 1, 6, 11, ... em
que cada nmero, com exceo do primeiro, igual ao anterior
mais cinco. Joana pra quando encontra o primeiro nmero de
trs algarismos. Que nmero esse?
29) Quantos nmeros de dois algarismos no so primos nem
mltiplos de 2, 3 ou 5?
30) Uma pra tem cerca de 90% de gua e 10% de matria slida.
Um produtor coloca 100 quilogramas de pras para desidratar at o ponto em que a gua represente 60% da massa total.
Quantos litros de gua sero evaporados? (Lembre-se: 1 litro
de gua tem massa de 1 quilograma.)
31) 2 melancias custam o mesmo que 9 laranjas mais 6 bananas;
alm disso, meia dzia de bananas custa a metade de uma melancia. Portanto, o preo pago por uma dzia de laranjas e uma
dzia de bananas igual ao preo de quantas melancias?
32) Contando-se os alunos de uma classe de 4 em 4, sobram 2.
Contando-se de 5 em 5, sobra 1. Sabendo-se que 15 alunos so
meninas e que, nessa classe, o nmero de meninas maior
que o nmero de meninos, qual o nmero de meninos dessa
classe?
33) So escritos todos os nmeros de 1 a 999 nos quais o algarismo
1 aparece exatamente 2 vezes (tais como 11, 121, 411 etc). Qual
a soma de todos estes nmeros?
34) Encontre dois nmeros, cada um com trs algarismos, usando
apenas os algarismos de 1 a 6, exatamente uma vez, de forma
que a diferena entre eles (o maior nmero menos o menor
nmero) seja a menor possvel.
35) Existem casas em volta de uma praa. Joo e Pedro do uma
volta na praa, caminhando no mesmo sentido e contando as
casas. Como no comearam a contar da mesma casa, a 5 casa

71

de Joo a 12 de Pedro e a 5 casa de Pedro a 30 de Joo.


Quantas casas existem em volta da praa?
36) Pintam-se de preto todas as faces de um cubo de madeira cujas
arestas medem 10 cm. Por cortes paralelos s faces, o cubo
dividido em 1.000 cubos pequenos, cada um com arestas medindo 1 cm. Determine:
a) O nmero de cubos que no possuem nenhuma face pintada de preto.
b) O nmero de cubos que possuem uma nica face pintada
de preto.
c) O nmero de cubos que possuem exatamente duas faces
pintadas de preto.
d) O nmero de cubos que possuem trs faces pintadas de preto.

A Revista da Olimpada
Brasileira de Matemtica.

37) Marcel leva exatamente 20 minutos para ir de sua casa at a escola. Certo dia, durante o caminho para a escola, percebeu que
esquecera em casa a revista Eureka, e queria voltar para casa a
fim de peg-la. Ele sabia que se continuasse a andar, chegaria
escola 8 minutos antes do sinal, mas se voltasse para pegar a
revista, no mesmo passo, chegaria escola com 10 minutos de
atraso. Que frao do caminho j tinha percorrido nesse ponto?
a)

2

5

b)

9

20

c)

d)

e)

38) Um elevador pode levar 20 adultos ou 24 crianas. Se 15 adultos


j esto no elevador, quantas crianas ainda podem entrar?
a) 4

b) 6

c) 8

d) 10 e) 12

39) Um copo est cheio de gua e seu peso de 520 gramas. Jogase

da gua fora. Assim, o peso cai para 440 gramas. O peso

do copo :
a) 80g

b) 120g

c) 130g

d) 250g

e) 960g

40) O Sr. Silva comprou um aparelho de TV, cujos canais variam


de 2 a 42. Cada toque no boto dos canais troca o canal para

72
seguinte. Ao chegar ao 42 voltar para o canal 2. Se ele est
assistindo o canal 15 e apertar o boto 394 vezes, em que canal
ir parar?
a) 25

b) 30

c) 35

d) 40

e) 45

41) Uma caixa dgua est com 400 litros de gua, corresponden2
do a
de sua capacidade. Num certo instante, uma torneira
5
aberta despejando, nessa caixa, 14 litros de gua por minuto.
Aps quantos minutos a caixa estar com

de sua capacidade?

a) 15

d) 18

b) 20

c) 30

e) 25

42) Seja N = 12 + 22 + 32 + 42 + + (196.883) 2 . Qual o algarismo


das unidades de N?
43) Um edifcio muito alto possui 1.000 andares, excluindo-se o
trreo. Do andar trreo partem 5 elevadores: o elevador A pra
em todos os andares; o elevador B pra nos andares mltiplos
de 5, isto , 0, 5, 10, 15, . . .; o elevador C pra nos andares mltiplos de 7, isto , 0, 7, 14, 21, . . .; o elevador D pra nos andares
mltiplos de 17, isto , 0, 17, 34, 51, . . . ; o elevador E pra nos
andares mltiplos de 23, isto , 0, 23, 46, 69, . . .
a) Mostre que, excetuando-se o andar trreo, no existe nenhum andar onde param 5 elevadores.
b) Determine todos os andares onde param 4 elevadores.
44) Joo comprou um livro e reparou que ele tinha 200 pginas.
Seu irmo mais novo arrancou, ao acaso, 25 folhas e somou os
nmeros das 50 pginas. Explique porque o resultado desta
soma no pode ser igual a 1998. Ateno: cada folha tem duas
pginas. A primeira folha tem as pginas 1 e 2; a segunda folha tem as pginas 3 e 4; e assim por diante.
45) Esmeralda, de olhos vendados, retira cartes de uma urna contendo inicialmente 100 cartes numerados de 1 a 100, cada um
com um nmero diferente. Qual o nmero mnimo de cartes que Esmeralda deve retirar para ter certeza de que o nmero de, pelo menos, um dos cartes seja um mltiplo de 4?

73

46) Na populao de 1.000 aves de uma espcie rara da floresta


amaznica, 98% tinham cauda de cor verde. Aps uma misteriosa epidemia que matou parte das aves com cauda verde,
esta porcentagem caiu para 95%. Quantas aves foram eliminadas com a epidemia?

Soluo dos problemas 1 a 46


1) Faamos uma lista:
0 : 00 1:11
2 : 22 3 : 33
4 : 44 5 : 55
11:11 22 : 22
Portanto, so 8 vezes.
2) 4 garrafas vazias = 1 garrafa cheia. Ento 43 = 4 10 + 3.
1 troca: 10 garrafas cheias (sobram 3 vazias). Consumidas as
10 garrafas, tem-se ento 13 garrafas vazias: 13 = 4 3 + 1.
2 troca: 3 garrafas cheias (sobra 1 vazia). Consumidas as 3
garrafas, temos, ento, 4 garrafas vazias.
3 troca: 1 garrafa cheia.
Total: 10 + 3 + 1 = 14 garrafas cheias.
3) Podemos escrever 25 = 2 + 23 e esta a nica maneira, pois 25
mpar e o nico primo par 2. Quaisquer outros dois primos
resultariam soma par.
4) Resolvemos o problema atravs de uma regra de trs simples:
25km

3
3 600
25 x = 3 600 x =
x = 72
25
600km x
Como cada litro custa R$ 0,75, sero gastos R$ 54,00
(72 0,75 = 54).
5) Sejam H a idade de Hlio e F a idade do filho, ambos h 18 anos.
Temos, ento, que h 18 anos, H = 3F e que H + 18 = 2 (F + 18).
Logo, F = 18 e H = 54. Hoje, F = 18 + 18 = 36 e H = 54 + 18 = 72.
Portanto, Hlio tem 72 anos e seu filho tem 36 anos.

74

6) Cada coluna corresponde a um resto na diviso por 9:


A 9k + 1
B 9k
C 9k + 2
D 9k + 8 E 9k + 3 F 9k + 7
G 9k + 4 H 9k + 6 I 9k + 5
Como 2.000 = 9 222 + 2, 2.000 est na coluna C.
7) As mdias das duas turmas melhoraram (item c). Isto quer dizer que
x1 + x2 + ... + x30 + x31 x1 + x2 + ... + x30
<
(I )
31
30
e que
x32 + x33 + ... + x61 x31 + x32 + ... + x61
<
( II )
30
31
Vamos verificar. Temos que:
30 ( x1 + ... + x30 ) + 30 x31 < 31 ( x1 + ... + x30 )
Logo, x1 + ... + x30 < 30 x31 . Esta ltima afirmao verdade,
pois x1 > x31 ,
, x30 > x31 . Logo, a primeira afirmao
verdadeira. A verificao da outra desigualdade anloga.
8) As bolas retiradas podem ser vermelhas ou brancas. Como s
existem 2 bolas brancas, pelo menos uma das bolas vermelha
(tem d).
9) Se foi o Mrio, dois garotos mentiram (Mrio e Carlos). Se foi o
Carlos, tambm dois garotos mentiram (Carlos e Pedro). Se foi
o Benjamin, temos trs mentirosos (Benjamin, Mrio e Carlos).
Logo, s pode ter sido o Pedro e, somente, Mrio mentiu.
10) O primeiro mltiplo comum de 7 e 8 56: 7, 8, 14, 16, 21, 24, 28,
32, 35, 40, 42, 48, 49, 56. At 56, inclusive, so 14 mltiplos. Posio 14 56, posio 28 56 2 = 112, ..., posio 98 56 7
= 392, posio 99 392 + 7 = 399, posio 100 392 + 8 = 400.
Portanto, o centsimo nmero 400.
11) | ______________ | ______________ | ______________ |
3a
4a
5a
6a
Entre a 3a e a 6a estaes a distncia de 3.300m, ou seja, entre duas estaes vizinhas a distncia de 3.300 3 = 1.100m.

75

Como so 12 estaes, teremos 11 destas distncias. Logo, 11


1.100 = 12.100m ou 12,1km.
12)

Quadrados perfeitos

16

25

36

49

64

81

Sucessores

17

26

37

50

65

82

Temos menos quadrados perfeitos de dois algarismos do que


primos de dois algarismos. Fazemos a contagem partindo dos
quadrados. So dois primos: 17 e 37.
13) Faamos uma contagem de hora em hora para as 12 primeiras
horas.
0h 1h : 2 vezes
6h 7h : 2 vezes
1h 2h : 2 vezes
7h 8h : 2 vezes
2h 3h : 2 vezes
8h 9h : 2 vezes
3h 4h :1 vez
9h 10h :1 vez
4h 5h : 2 vezes
10h 11h : 2 vezes
5h 6h : 2 vezes
11h 12h : 2 vezes
Temos, no total, 44 vezes.
14) Quantas caixas de dimenses 20 20 20 cabem em um caixote de 80 120 60?
Na largura: 4 20 = 80 4 na largura
No comprimento: 6 20 = 120 6 no comprimento
Na altura: 3 20 = 60 3 na altura
Portanto, cabem 4 6 3 = 72 caixas cbicas no caixote. Logo,
podemos acomodar 576 (72 8) latas de palmito por caixote.
15) Vamos decompor todos em fatores primos e verificar qual o
maior.
345 = 339 36

920 = 340 = 339 3

2714 = 342 = 339 33

2439 = (3.8)39 = 339.2117


Logo, 2439 o maior.

76

16)

( x 3) + 12
= 15 3 x + 12 = 105
7
3 x = 105 12
3 x = 93 x = 31

17) So 2 segundos de diferena por volta. Logo, em 80 voltas so


160 segundos. Em 160 segundos, Carlinhos d 5 (160 32) voltas. Portanto, quando Carlinhos estiver completando a volta de
nmero 80, Paulinho estar completando a volta de nmero 75
(80 5).
18) 997 979 799, 988 898 889. Portanto, so seis nmeros.
19) 11:20 12:20 1 real
12:20 15:20 0,75 3 = 2,25
15:20 15:40 0,75
Total: 1 + 2,25 + 0,75 = 4 reais
20) So 90 os nmeros de dois algarismos, sendo 45 pares e 45
mpares.
Impares: 11, 13, 15, . . . , 97, 99
Pares: 10, 12, 14, . . . , 96, 98
A diferena entre dois nmeros da mesma coluna 1. Como
so 45 colunas, teremos que a diferena 45.
21) J : idade de Joo
P : idade de Pedro
C: idade de Carlos
A: idade de Antnio
Temos que: J > P , C > P , A > C , J > C e A > J.
Colocando em ordem decrescente: A > J > C > P , ou seja, Pedro o mais jovem.
22) A rea da fazenda calculada assim: 10 20 = 200 km2. Agora,
200 km2 200 = 1 km2 = 1.000.000 m2. Logo, cada famlia recebeu 1.000.000 m2.

77

23)

Acar

Manteiga

Leite

Farinha

Quantidade

100g

50g

400g

12 bolinhos

500g

300g

5.000g

512 = 60
bolinhos

1005=500

505=250

1
5
5 =
2
2

4005=2.000g

A resposta 60 bolinhos.
24) A equao do problema 3 p + 5r = 97 , em que p o nmero
de primos e r, o nmero de rodas. Analisando as alternativas
a) 19 5 = 95 e 97 95 = 2, que impossvel (pois uma pessoa
tem 3 pernas e um carro tem 5 rodas).
b) 165 = 80. Mas, 97 80 = 17, que no mltiplo de 3. Agora,
17 5 = 85 e 97 85 = 12, que mltiplo de 3. As outras possibilidades no podem se concretizar (18 5 = 90, mas 97
90 = 7, que no mltiplo de 3. E 19 5 = 95, mas 97 95 = 2
que no mltiplo de 3). Logo, o nmero mximo de carros
17. Essa uma alternativa correta.
c) 9 3 = 27, 14 5 = 70 e 27 + 70 = 97. Ou seja, esses nmeros
so possveis, mas no podemos afirmar que s estes nmeros funcionam, pois poderamos ter tambm 17 carros e 4
pessoas.
d) de acordo com (b), possvel existir 17 carros.
e) 5 2 = 10, 3 29 = 87 e 87 + 10 = 97. Isto , podem existir 2
carros e 29 pessoas. Esta no uma alternativa correta.
25) Seja x o nmero de moedas de 1 real e y, o nmero de moedas
x + 0,5 y = 76
y = 48

de 50 centavos.
. Logo, a diferena
x + y = 100
x = 52
de 4 moedas.
26)

8+2+4+ x+6+ y
= 5 ( y + 2 + 4 + 6) + 8 + x = 30 . Assim,
6
y+2+4+6
= 3 y + 2 + 4 + 6 = 12 y = 0 . Logo, x = 10.
4

78

27) Note que, para isso, o primeiro algarismo do nmero par deve
ser sucessor do primeiro algarismo do nmero mpar e, alm
disso, os dois algarismos finais de cada um devem ser o menor
e o maior possvel, respectivamente. Assim, podemos tomar
os nmeros 402 e 397 ou 602 e 597. A diferena, em ambos os
casos, 5.
28) 1, 6, 11, 16, . . . , n, n + 5, . . . Todos os nmeros deixam resto 1
na diviso por 5. O menor nmero de 3 algarismos 100, que
deixa resto zero. Logo, dever ser o 101, que deixa resto 1.
29) Entre 10 e 99, temos 90 nmeros com dois algarismos.
Primos: 11 13 17 19 23 29 31 37 41 43 47 53 59 61 67 71 73 79
83 89 97, totalizando 21 primos.
Mltiplos de 2: 45 nmeros
Mltiplos de 3: 30 nmeros
Mltiplos de 5: 18 nmeros
Mltiplos de 6: 15 nmeros
Mltiplos de 10: 9 nmeros (mltiplos de 5 e 2)
Mltiplos de 15: 6 nmeros (mltiplos de 5 e 3)
Mltiplos de 30: 3 nmeros (mltiplos de 5, 3 e 2)
Logo, entre 10 e 99, existem 45 mltiplos de 2, 15 mltiplos de
3, mas que no so mltiplos de 2, mais 6 (= 18 [9 + 6 3])
mltiplos de 5, que no so mltiplos nem de 2, nem de 3.
Logo, existem 3 nmeros que satisfazem o problema: 7 7 = 49,
7 11 = 77 e 7 13 = 91.
Outra forma (mais direta): os nmeros procurados so produtos de primos e estes primos no podem ser 2, 3 e 5. Restam
7 7 = 49, 7 11 = 77 e 7 13 = 91.
30) Sejam A a quantidade de gua e MS a quantidade de matria
slida. Ento 90%A + 10%MS = 1 pra.
100kg = 90kgA + 10kgMS
Massa total = 60%A + 40%MS

79

Observando que a massa slida no evapora, temos:


40% 10kg
100% 25kg
60% 15kg
Ou seja, restam 15kg de gua depois de desidratar. Logo, evaporam-se 90 15 = 75kg, ou 75 litros de gua.
31) Sejam m, b e l o nmero de melancias, bananas e laranjas, respectivamente. Temos que 6b + 9l = 2m . Alm disso, 1m = 12b.
Assim, 1l = 2b . Logo, 12 bananas mais 12 laranjas correspondem ao preo de 3 melancias.
32) Seja n o nmero de alunos. Ento, n = 4x + 2e e n = 5 y + 1 . Temos
que 29 = 5.5 + 4, 28 = 5.5 + 3, 27 = 5.5 + 2, 26 = 5.5 + 1 = 4.6 + 2.
Logo, so 26 alunos: 15 meninas e 11 meninos.

33) Os nmeros, nesse intervalo, so da forma x11, 1x1 ou 11x (no


primeiro caso, se x = 0, o nmero 11). Agora, x11 + 1x1 + 11x = x102 + 1.10 + 1
x11 + 1x1 + 11x = x102 + 1.10 + 1 + 1.102 + x.10 + 1 + 1.102 + 1.10 + x , que igual
a ( x + 2).102 + ( x + 2).10 + ( x + 2) , que igual a 111.( x + 2) . Fazendo x variar de 0 a 9, x 1 , temos que a soma 222 + 444 +
555 + 666 + 777 + 888 + 999 + 1110 + 1221 = (2 + 4 + 5 + 6 + 7 +
8 + 9 + 10 + 11). 11 = 6882.
34) Devemos subtrair o maior possvel do menor possvel: 234
165 = 69, 314 265 = 49, 412 365 = 47, 512 463 = 49, 612 543
= 69. Portanto, os nmeros so 412 e 365.
35) Da 5 para a 12 de Pedro so 7 casas (inclusive a 12). A 5a casa
de Joo seria tambm a sua 37. Ou seja, 32 (= 37 5) a ltima
casa de Joo. Logo, so 32 casas em volta da praa.
36) a) 512; b) 384; c) 96;

d) 8

37) O tempo necessrio para Marcel retornar sua casa e depois


fazer todo o percurso at o ponto onde estava de 18 minutos, pois chegaria adiantado em 8 minutos, mas acabou mesmo
chegando com 10 minutos de atraso. Portanto, levou 9 minutos

80
para ir de sua casa at o ponto de retorno, o que corresponde a
9
da distncia de sua casa ate a escola.
20
38) A carga total do elevador de 20 adultos, mas j se encontram no elevador 15 adultos, o que corresponde a
restando

da carga,

da carga para completar com crianas. Como o

elevador pode levar 24 crianas e

de 24 6, podem entrar

ainda 6 crianas no elevador.


39) Note que

da gua que foi jogada fora corresponde dife-

rena 520 440 = 80. Logo,


gramas. Tem-se que

da gua do copo cheio pesa 80

da gua ser 400 gramas e o peso do

copo ser a diferena 520 400 = 120. Portanto, o peso do copo


de 120 gramas.
40) Um aparelho de TV tem 41 canais. Divida 394 pelo nmero de
canais, ou seja, 41. Na diviso, obteremos resto igual a 25. Agora, basta somar 25 ao nmero do canal que o Sr. Silva assistia,
ou seja, 15. Obteremos, como resultado, 40. Logo, ele parou no
canal 40.
2
da capacidade da
5
correspondem a 1.000 litros. preciso deixar a

Sabemos que 400 litros correspondem a


caixa. Ento
caixa com

de sua capacidade. Como

de 1.000 750 e a

caixa j tinha 400 litros, restam 350 litros para colocar na caixa.
Agora, basta dividir 350 por 14 e obteremos 25. Logo, em 25
minutos a caixa estar com

de sua capacidade.

41) Os algarismos das unidades dos quadrados dos nmeros de 1


a 10 so, respectivamente, 1, 4, 9, 6, 5, 6, 9, 4, 1 e 0. Ora, a soma
dos nmeros formados por esses algarismos 45. Portanto, a
soma 12 + 22 + 32 + 42 + 52 + 62 + 72 + 82 + 92 + 102 tem, como al-

81

garismo das unidades, o dgito 5. De 11 a 20, os algarismos das


unidades dos nmeros se repetem na mesma ordem. Portanto,
o algarismo das unidades da soma de seus quadrados tambm
5. Conseqentemente, a soma dos quadrados dos nmeros de
1 a 20 tem 0 como algarismo das unidades. Logo, a soma 12 + 22
+ 32 + 42 + . . . + n2 tem zero como algarismo das unidades se N
mltiplo de 20. Como N = 12 + 22 + 32 + 42 + . . . + (196.883)2 =
12 + 22 + 32 + 42 + . . . + 196.8802 + 196.8812 + 196.8822 + 196.8832,
conclumos que o algarismo das unidades de N o mesmo do
nmero 0 + 1 + 4 + 9 = 14, ou seja, 4.
42) a) O elevador B pra nos mltiplos de 5; o elevador C pra nos
mltiplos de 7; o elevador D pra nos mltiplos de 17; o elevador E pra nos mltiplos de 23. Como 5, 7, 17 e 23 so nmeros
primos, para que todos parem num mesmo andar, este tem
que ser mltiplo de 571723 = 13.685. Mas o prdio s tem
1.000 andares.
b) Para que num andar parem exatamente quatro elevadores,
devem parar A, que pra em todos, e os trs restantes. Agora,
B, C e D param nos mltiplos de 5717 = 595; B, C e E param
nos mltiplos de 5723 = 805; B, D e E param nos mltiplos de
51723 = 1.955; C, D e E param nos mltiplos de 71723 = 2.737.
Logo, os andares em que param 4 elevadores so o 595 e 805.
43) Cada folha tem duas pginas tais que a soma dos seus respectivos nmeros mpar. Ao somarmos todos esses 25 nmeros,
obteremos, necessariamente, uma soma mpar que, portanto,
no pode ser igual a 1.998.
44) De 1 a 100, existem 25 cartes mltiplos de 4. Logo, 75 cartes
no contm mltiplos de 4. No pior caso possvel, Esmeralda
tiraria todos esses cartes antes de sair algum carto com mltiplo de 4. Assim, para ter certeza de que pelo menos um dos
nmeros tirados seja mltiplo de 4, Esmeralda deve tirar todos
aqueles, mais um, ou seja, 76 cartes.
45) Inicialmente, existiam 980 (98% de 1.000) aves de cauda verde e
20 (2% de 1.000) das demais. Aps a epidemia, estas 20 aves correspondem a 5% do total, que agora de 400 aves, das quais 380
so de cauda verde. Portanto, morreram 980 380 = 600 aves.

82

3.6 Problemas Propostos


1) Qual o resto da diviso de
111.111.111.111.111.111.111.111.111.111 por 16?

(OBM/96/F1)

2) Qual o dgito das unidades de 31998?

(OBM/98/F1)

As solues dos problemas


encontram-se nas revistas
Eureka!, disponveis no stio
da Olimpada Brasileira de
Matemtica: http://www.
obm.org.br.

3) Quantos so os nmeros inteiros x que satisfazem 3 < x < 7?



(OBM/98/F1)
4) Em um conjunto de pontos do espao, a distncia entre dois
pontos quaisquer igual a 1. Qual o nmero mximo de pontos que pode haver neste conjunto?
(OBM/98/F1)
x + 99
x + 19
(OBM/99/F1)

5) Quantos so os possveis valores inteiros de x para que


seja um nmero inteiro?

n
, se n
2
par e f (n) = 3n + 1 , se n mpar. Qual o nmero de solues
de f ( f ( f (n))) = 16 ?
(OBM/99/F1)

6) Para todo nmero natural n, definimos a funo: f (n) =

7) Em um aqurio h peixes amarelos e vermelhos. 90% so


amarelos e 10% so vermelhos. Uma misteriosa doena matou
muitos peixes amarelos mas nenhum vermelho. Depois que
a doena foi controlada, verificou-se que no aqurio 75% dos
peixes vivos eram amarelos. Aproximadamente, que porcentagem dos peixes amarelos morreram?
(OBM/99/F1)

8) Na circunferncia ao lado, o raio 1, o arco AB mede 70 e o


arco BC mede 40. Qual a rea da regio limitada entre as
cordas AB e AC?

(OBM/99/F1)
9) Dois irmos herdaram um terreno ABC com a forma de um
tringulo retngulo em A e com o cateto AB de 84m de comprimento. Eles resolveram dividir o terreno em duas partes de
mesma rea por um muro MN paralelo a AC, como mostra a
figura 3.25. Qual o valor aproximado de BM? (OBM/99/F1)

B
A
Figura 3.24

83
B

M
A

C
Figura 3.25

10) Se 0 < x < 90 e cos x = 1/4, ento x est entre:


a) 0 e 30

b) 30 e 45

d) 60 e 75

e) 75 e 90

c) 45 e 60
(OBM/99/F1)

11) Um quadrado ABCD possui lado igual a 40cm. Uma circunferncia contm os vrtices A e B e tangente ao lado CD. Qual
o raio da circunferncia?
(OBM/99/F1)
12) O nmero

igual a:

a) 0,2222... b) 0,3333... c) 0,4444... d) 0,5555... e) 0,6666...



(OBM/99/F1)
13) Seja f uma funo definida para todo x real, satisfazendo as
condies:
f ( x + 3) = f ( x) f (3)
Determine o valor de f (3) = 2 .

(OBM/97/Junior/F1)

14) Qual o nmero de pares (x, y) de nmeros reais que satisfa2


2
x xy y + 1 = 0
zem o sistema 3 2
?
2
x x y xy + x y + 2 = 0

(OBM/97/Junior/F1)
15) Qual o nmero de pares ( x, y ) de nmeros inteiros que satisfazem a equao x + y + xy = 120 ?
(OBM/97/Snior/F1)
16) Temos 10.000 fichas iguais com a forma de um tringulo eqiltero. Com estes pequenos tringulos se podem formar hexgonos regulares sem superposio de fichas ou vazios. Considere

84

agora o hexgono regular que desperdia a menor quantidade


de tringulos. Quantos tringulos sobram? (III Olimpada de
Maio/N2)
17) Se seu salrio sobe 26% e os preos sobem 20%, de quanto aumenta seu poder aquisitivo?
(OBM/97/Snior/F1)
18) Qual o nmero de solues reais da equao x 2 = 2 x ?

(OBM/97/Snior/F1)
19) Quantos so os nmeros de sete algarismos que so mltiplos
de 388 e terminam em 388?
(Olimpada de Maio/97/N2)
20) A funo f associa a cada nmero real x o menor elemento do
(15 x)

conjunto x + 1,
. Qual o valor mximo de f ( x) ?
2


(OBM/98/F1)
21) Qual a soluo da inequao

x+

1
2?
x

(OBM/98/F1)

22) Determine o maior nmero natural n para o qual existe uma r


ordenao (a, b, c, d ) de (3, 6,9,12) (isto , {a, b, c, d } = {3, 6,9,12})
tal que o nmero n 3a.6b.9c.12d seja inteiro. Justifique sua resposta.
(OBM/99/F2)
23) Nos extremos do dimetro de um crculo, escreve-se o nmero 1 (primeiro passo). A seguir, cada semicrculo dividido ao
meio e, em cada um dos seus pontos mdios, escreve-se a soma
dos nmeros que esto nos extremos do semicrculo (segundo passo). A seguir, cada quarto de crculo dividido ao meio
e, em cada um dos seus pontos mdios, coloca-se a soma dos
nmeros que esto nos extremos de cada arco (terceiro passo).
Procede-se assim, sucessivamente. Sempre cada arco dividido
ao meio e, em seu ponto mdio, escrita a soma dos nmeros
que esto em seus extremos. Determinar a soma de todos os
nmeros escritos aps 1.999 passos.
(OBM/99/F2)
24) Determine todos os inteiros positivos n para os quais possvel montarmos um retngulo 9 10, usando peas 1 n.

(OBM/99/F2)

85

Referncias
[1] MEGA, Elio; WATANABE, Renate. Olimpadas brasileiras de matemtica: 1. a 8.: problemas e resolues. So Paulo: Ncleo, 1988.
[2] MOREIRA, Carlos et al. Olimpadas brasileiras de matemtica: 9. a
16.: problemas e resolues. Rio de Janeiro: SBM, 2003.
[3] EUREKA!. Rio de Janeiro: SBM. Disponvel em: <www.obm.org.
br/opencms/revista_eureka>. Acesso em: 29 mai. 2009.
[4] OLIMPADAS Brasileiras de Matemtica. Disponvel em: <www.
obm.org.br/opencms/>. Acesso em: 29 mai. 2009.
[5] OLIMPADAS Brasileiras de Matemtica nas Escolas Pblicas.
Disponvel em: <www.obmep.org.br>. Acesso em: 29 mai. 2009.
[6] OLIMPADAS Regionais de Matemtica. Disponvel em: <www.
orm.mtm.ufsc.br>. Acesso em: 29 mai. 2009.

Captulo 4
Raciocnio Lgico

89

Captulo 4
Raciocnio Lgico
4.1 Introduo
HOUAISS, A. Dicionrio
Houaiss da Lngua
Portuguesa. Rio de Janeiro:
Objetiva, 2001.

Matemtica uma cincia que lida com abstraes e, para lidar


com elas sem nos perder na complexidade da mente humana,
precisamos de lgica. Lgica uma palavra latina que se origina,
segundo Houaiss, da reduo da expresso grega logik tkhne,
que significa arte de raciocnio. A matemtica moderna apresentada como cincia ao pblico especializado segundo um mtodo chamado de mtodo axiomtico. Esse mtodo baseia-se em
cadeias ascendentes de raciocnios, cuja base so os axiomas, que
so proposies sobre conceitos primitivos, aceitas como verdades absolutas sobre esses conceitos. Todas as afirmaes que aparecem nas cadeias compem aquilo que chamamos de uma teoria matemtica. Por exemplo, geometria euclidiana plana uma
teoria matemtica que comea com postulados sobre conceitos
primitivos como ponto, reta, passar por, estar entre e congruente.
Um exemplo de axioma por dois pontos passa uma nica reta;
outro, o ponto C est entre os pontos A e B.
Os conceitos primitivos podem ser interpretados de formas diversas, cada forma dita um modelo da teoria. Por exemplo, a
geometria analtica plana um modelo para a geometria euclidiana plana, na qual ponto um par ordenado
; reta um
conjunto de pontos que satisfazem uma equao linear; passar
por dois pontos significa conter esses dois pontos como elementos do conjunto; um ponto C estar entre dois pontos A e B, numa
reta, significa que as coordenadas de C satisfazem a equao
t ( x A , y A ) + (1 t ) ( xB , yB ) , para algum nmero real
; segmentos
congruentes significam segmentos de mesma norma.
No Ensino Fundamental, basicamente, falamos do alfabeto e da
sintaxe na matemtica. Queremos dizer, com isso, que aprendemos
a escrever algumas expresses, equaes, desigualdades e proposies matemticas mais abrangentes, com o alfabeto da matemtica,

90

que uma mistura de nmeros, letras latinas e gregas etc. Por exemplo, a equao (a + b) 2 = a 2 + b 2 + 2ab uma orao matemtica que
estabelece que a 2 + b 2 + 2ab um sinnimo da palavra (a + b) 2 .
No Ensino Mdio, a nfase comea a se voltar para a semntica, ou
seja, para o valor de verdade de sentenas matemticas. Um teorema uma sentena matemtica verdadeira. A negao do teorema
uma sentena falsa. Dissemos que, no mtodo axiomtico, as sentenas iniciais da teoria os axiomas so verdadeiras. Como se criam
novas afirmaes verdadeiras (teoremas) a partir dos axiomas?
isso que vamos tratar aqui, de modo muito informal. Gostaramos
que voc, aluno, compreendesse que criamos novas sentenas matemticas a partir de conectivos. So cinco os conectivos: e, ou, no
(~), se ... ento (), se e somente se (). Dada uma afirmao do
tipo se A ento B (ou, em smbolos, A B), dizemos que A a hiptese e B, a tese. A recproca de A B , por definio, a proposio
B A; a contra-positiva de A B a proposio ~B ~A; a sua
contra-recproca ~A ~B.
Alguns teoremas do clculo proposicional so:
1) Se A B e B C, ento, A C.
2) A B e ~B ~A so equivalentes (tm a mesma tabela de
verdade).
Tabelas de Verdade Bsicas
A

AeB

A ou B

~A

Conjunes que conectam


sentenas antigas ou
advrbio, para formar uma
nova.

91

3) ~(A e B) e (~A ou ~B) so equivalentes (tm a mesma tabela de


verdade).
4) ~(A ou B) e (~A e ~B) so equivalentes (tm a mesma tabela de
verdade).
5) A B e ~A ~B so equivalentes (tm a mesma tabela de
verdade).
6) ~(A B) e (A e ~B) so equivalentes (tm a mesma tabela de
verdade).
Primeiro, abordaremos o clculo proposicional com textos cotidianos. Nessas proposies no aparecem quantificadores, que so dois:
o existencial (denotado por , que lido como existe) e o universal
(denotado por , que se l como para todo). Depois, introduzimos
frases coloquiais com quantificadores.

4.2 Mtodo axiomtico


A matemtica moderna apresentada segundo um mtodo chamado de mtodo axiomtico. Este mtodo baseado em cadeias de
raciocnios a partir de axiomas, que so proposies sobre alguns
conceitos iniciais as quais aceitamos como verdades absolutas.
Podemos considerar que proposies so como perodos em gramtica. Podemos ter perodos simples, que so formados por uma s orao. Se essa orao for fechada, no sentido que no h algo indefinido, dizemos que ela uma proposio atmica. Por exemplo, Rumi
um poeta do amor exttico uma orao fechada, enquanto que
X um poeta do amor exttico uma orao aberta. Eu posso por,
no lugar de X, algum nome e a frase ser verdadeira ou falsa, dependendo, obviamente, do nome da pessoa que eu puser. Por exemplo,
substituindo-se X por Augusto de Campos, obtemos uma sentena
falsa, pois Augusto de Campos um poeta abstrato, que lida com o
aspecto grfico das letras nas palavras, e a maior parte de sua criao
literria no se refere explicitamente ao xtase no amor (no entanto,
ele traduziu John Donne, poeta do amor devoto). Outra frase fechada a seguinte: H polticos que no enganam seus eleitores. Apesar de no se referir a ningum em particular, pode-se verificar se
essa frase ou no verdadeira no conjunto de todos os polticos que

92

vivem no planeta. Apesar desse perodo no ser simples, considerase essa frase como uma sentena atmica, pois a orao que no
enganam seus eleitores funciona como um adjetivo.
Uma proposio do tipo Rumi um poeta do amor exttico e sua
morte comemorada todo dia 17 de dezembro na cidade turca de
Knia uma proposio que conecta a proposio acima, via conjuno e, proposio todo ano x, a morte de Rumi comemorada
na cidade turca de Knia, no dia 17 de dezembro de x. Aquela no
uma proposio atmica, pois seu valor de verdade depende do
valor de verdade de cada sentena que a compe. Nesse caso, aquela frase ser verdadeira se, e somente se, cada frase for verdadeira,
independentemente da outra. Por exemplo, se em 2006 no se comemorar a sua morte no dia 17 de dezembro, toda a sentena ser
falsa. Uma proposio que no atmica corresponde a um perodo
gramatical composto por vrias oraes.
No estudo abstrato das proposies, usamos letras para representar
uma frase. Por exemplo, a proposio composta acima poderia ser
expressa como E ( R ) (x) M ( x, R ) , em que E ( X ) a relao X
um poeta do amor exttico e M ( x, X ) a relao a morte de X
comemorada na cidade de Knia no dia 17 de dezembro de x.
Assim, as proposies atmicas correspondem a expresses simblicas com smbolos relacionais (que representam uma relao), smbolos funcionais (para representar funes), smbolos para variveis
(em geral, do final do alfabeto: x, y, z), smbolos para constantes (em
geral, do incio do alfabeto: a, b... ; ou algarismos: 1, 2...), conectivos e
quantificadores. Reciprocamente, podemos criar expresses simblicas, seguindo uma gramtica prpria, tais que possam ser traduzidas para proposies em linguagem corrente, uma vez que se faa
uma correspondncia entre smbolos e palavras ou entre smbolos e
frases. Isso o que se chama de interpretao. E o conjunto no qual
se faz essa interpretao se chama de modelo. Em um modelo, faz
sentido atribuir um valor de verdade: ou V (verdadeiro) ou F (falso).
Voltemos ao mtodo axiomtico. Partindo-se de um conjunto de proposies iniciais, criamos proposies novas, utilizando conectivos.
Em matemtica, usamos 5 conectivos, que aparecem como e, ou,
se...ento (ou implica em, ou somente se), se e somente se (ou

93

equivale a) e no. Essas novas proposies tero algum valor de


verdade, supondo-se que o valor de verdade atribudo s proposies iniciais V. Em matemtica, dada uma proposio que fale sobre os conceitos iniciais (ou sobre outros definidos a partir desses),
o desafio montar uma proposio composta, como uma cadeia de
proposies, tal que o ltimo elemento dela seja a proposio dada.
Essa cadeia uma prova ou demonstrao. A proposio dada
dita um teorema se for possvel esse encadeamento de proposies
que cheguem nela. Enquanto no for engendrada essa cadeia, essa
proposio dita uma conjetura.
Consideremos uma proposio simblica Q. Por exemplo, suponha
que Q a proposio escrita como A (( B C ) D) . Uma tabela
com os possveis valores de verdade da proposio Q, a partir dos
possveis valores de verdade de cada proposio atmica que a compe (A, B, C, D), chama-se a tabela de verdade de Q.
H proposies cujas tabelas de verdade apresentam o valor final
V (verdadeiro), no importando os valores de verdade das proposies atmicas que a compem. Essas proposies so chamadas
de tautologias. Seja A uma proposio qualquer formada a partir de
proposies atmicas (mais conectivos). Verificar se A ou no uma
tautologia o que se chama Clculo Proposicional (ver 4.2).
Observe que existem frases para as quais no possvel decidir se
so verdadeiras ou falsas. Por exemplo, a frase x um nmero inteiro primo pode ser verdadeira ou no. Isso ocorre porque no nos
referimos a algum x especfico. Para decidir se esta frase verdadeira ou falsa precisamos especificar o valor de x. Frases como estas
chamamos de frmulas abertas. Uma frase ou frmula fechada
algo como 3 um nmero primo.
Note que, se o conjunto dos nmeros inteiros, a proposio
existe x em , tal que x primo verdadeira, assim como, para
cada x em , x2 no um nmero primo. Aqui, deparamo-nos com
dois elementos novos, que so o quantificador existencial existe
e o quantificador universal para todo (ou para cada). Essas frmulas com quantificadores tambm so ditas fechadas: cada quantificador limita o alcance da varivel. Frmulas fechadas so ditas
sentenas (ver 4.3).

94

4.3 Clculo Proposicional


Seja M um conjunto de letras latinas maisculas, prximas ao incio
do alfabeto, com ou sem subscritos A, B12, C' etc. Vamos chamar
os elementos de M de smbolos proposicionais, ou de smbolos relativos (ou relacionais) de ordem zero. O que faremos a seguir ,
intuitivamente, considerar os smbolos de M como proposies atmicas que, com o auxlio dos conectivos ~ (negao), (conjuno),
(disjuno), (implicao) e (equivalncia) so combinadas
de tal maneira a formarem proposies mais complexas, cujos valores verdade (verdadeiro ou falso) so determinados pelos valores
de verdade (verdadeiro ou falso) atribudos s proposies atmicas
que as compem.
Seja
Definio. Uma seqncia finita de smbolos de S uma proposio
de S se, e s se, a seqncia obtida a partir de um nmero finito de
aplicaes das seguintes regras:
i) um smbolo proposicional uma proposio de S.
ii) Se X uma proposio, ento
iii) Se X e Y so proposies, ento
so proposies de S .

uma proposio de S.
,

, (X Y) ,

Exemplos:
i) (( A B) (( B) ( (C D)))) uma proposio.
ii)

, porm, no uma proposio, pois no obtida por um nmero de aplicaes das regras dadas acima.

Seja f uma funo de S em {V , F } . Se


e f ( B) = F , ento
dizemos que A tem um valor verdade V (verdadeiro) e que B tem o
valor de verdade F (falso). Chamamos f de uma atribuio de valores
de verdade aos smbolos de S. Agora, vamos estender f ao conjunto
de todas as proposies de S. Seja X uma proposio contida nesse
conjunto. O valor de verdade de X fica ento determinado pelas seguintes regras:

95

i) se X = ( Y ) ento o valor de verdade de X V se e s se o de


Y F.
ii) se
ento o valor de verdade de X V se e s se o
valor de verdade de ambas as proposies Y e Z V.
iii) se
ento o valor de verdade de X V se e s se o
valor de verdade de Y V ou, se no, o valor de verdade de Z V.
iv) se
ento o valor de verdade de X V se e s se o
valor de verdade de Y F ou, se no, o valor de verdade de Z V.
v) se
ento o valor de verdade de X V se e s se ambas as proposies Y e Z tm o mesmo valor de verdade.
Seja { A1 , ... , An } o conjunto dos smbolos proposicionais que ocorrem em X. Para cada atribuio de valores verdade a { A1 , ... , An } obtemos, ento, um nico valor de verdade para X. Assim, cada proposio determina uma funo que tem tantos argumentos quantos
forem os smbolos proposicionais distintos que aparecem na proposio. Esta funo pode ser graficamente representada por uma
tabela tabela de verdade da proposio. Por exemplo, a proposio
(( A B) ( B)) tem a seguinte tabela de verdade:

(( A B) ( B))

Observemos os seguintes exemplos de tabelas de verdade:


i) ( A ( B A))

( A ( B A))

96

ii)
( A B) ^ ( B C )

iii) (( ( A) A))
A

( A)

( ( A))

(( ( A) A))

Definio. Uma proposio como as dos exemplos acima, cujo valor


de verdade sempre V no importando que valores de verdade so
atribudos aos smbolos que a compem, chamado de tautologia.
Dizemos que duas proposies X e Y so equivalentes se e s se
uma tautologia.
Observaes:
i) Se numa proposio existem exatamente n smbolos proposicionais ( n 1 ), ento existem 2n atribuies possveis de valores de verdade a esses smbolos e, logo, 2n linhas na nsua tabela
2
de verdade. Assim, podemos obter no mximo 2 proposies distintas, a menos de equivalncias, a partir de n smbolos proposicionais, pois para cada proposio construda a
partir desses smbolos, existem, no mximo, 2n atribuies,
nas quais a proposio ou tem valor de verdade V ou tem valor
de verdade F.

97

ii) Para cada proposio X que contm exatamente m smbolos


proposicionais, existem muitas proposies com n smbolos
proposicionais, n > m , equivalentes a X. Por exemplo,

equivalente a (( A B ) (C C )) .
At aqui vimos que S se constitui de um conjunto de smbolos a
partir dos quais formamos certas expresses as proposies. Um
sistema formal definido quando, alm de um conjunto de smbolos
e de regras de formao de sentenas (no caso do Clculo Proposicional, proposies), existem regras de transformao com as quais
as sentenas especiais, os teoremas, so criadas. As regras de transformao so:
i) um conjunto de sentenas iniciais, ditas axiomas;
ii) regras de inferncia, pelas quais outros teoremas so criados,
a partir dos axiomas.
Sejam X, Y, Z proposies de S. Ento as seguintes proposies de S
so axiomas:
i) X (Y X )
ii) ( X (Y Z )) (( X Y ) ( X Z ))
iii) (( Y ) ( X )) ((( Y ) X ) Y )
iv) ( X Y ) (( X Y ) (Y X ))
v)
vi) ( X Y ) (( X ) Y )
vii) ( ( X Y )) (( X ) ( Y ))
Definio. Uma proposio de S um teorema de S se e s se obtida por um nmero finito de aplicaes das seguintes regras:
i) um axioma um teorema de S;
ii) se X e ( X Y ) so teoremas de S ento Y um teorema de S.
Desse modo podemos falar do sistema formal S.
Considere, agora, um conjunto de proposies K e seja Y uma proposio de S.

98

Definio. Y deduzvel de K, em smbolos, K Y , se e s se Y


um teorema de S ou, se no, existem proposies X 1 ,..., X n em K tais
que
um teorema de S.
Definio. K consistente se e s se existe uma proposio Y de S tal
que Y no deduzvel de K.
possvel mostrar que se K um conjunto de proposies consistente, ento existe uma atribuio de valores verdade aos smbolos
proposicionais que ocorrem nas proposies de K tal que o valor
de verdade correspondente a cada proposio de K V. Uma conseqncia disto que se X uma tautologia, ento X um teorema de
S (a recproca bvia).

4.4 Frmulas abertas


Definio. Uma frmula aberta uma expresso S ( x) tal que, interpretada em um conjunto A, ou S (a ) verdadeira ou S (a ) falsa,
para cada elemento a A .
Exemplo: Abaixo esto algumas frmulas abertas, sobre os nmeros naturais.
x + 1 > 8
x + 5 = 9
x primo
x um divisor de 10
x + 1 < 1
Definio. O conjunto verdade de uma frmula aberta em A o
conjunto de todos os elementos a A tais que S (a ) uma sentena
verdadeira.
Denotando por VS o conjunto verdade de S em A, temos que

Vs = {a A tal que S (a ) verdadeira} ,


ou, simplesmente,

Vs = {a A | S (a )} .

99

Exemplo: O conjunto verdade de x + 1 > 8


em Vs = {x | x + 1 > 8} = {8,9,10,...}

Exerccio
Determine o conjunto verdade das seguintes frmulas:
a) x + 5 = 9 , em ;
b) x um divisor de 10, em ;
c) x mltiplo de 3, em (o conjunto dos nmeros inteiros);
d) x 2 7 x + 12 = 0 , em (conjunto dos nmeros reais);
e) 3 x 2 > 5 , em P (conjunto dos nmeros inteiros pares);
f) x 2 + 1 < 2 , em ;
g)

x 2 + 1 = x 1 , em .

Observao:
1) Note que o conjunto verdade da frmula aberta x 2 0 em
o prprio , ou seja, x 2 0 uma sentena verdadeira para
TODO x em .
2) No item c) acima, nem todo nmero natural torna a sentena
verdadeira e, portanto, o conjunto verdade no todo o conjunto . Porm, EXISTEM alguns nmeros naturais que tornam a sentena verdadeira, a saber, 3, 6,9...
, ...

4.5 Quantificao
O quantificador existencial, que denotado por , est associado a
palavras como para algum e existe(m).
Por exemplo, a sentena Existe um nmero inteiro que maior que
zero pode ser reescrita, simbolicamente, por
. A negao desta sentena No existe um nmero inteiro maior que zero,
que tem o mesmo valor de verdade que Todos os nmeros inteiros
so menores ou iguais a zero, ou, ainda, (x ) x 0 .

100

O quantificador universal, que denotado por , est associado a


palavras como todo ou qualquer ou cada.
Assim, se S ( x) uma frmula aberta em um conjunto A,

((x A) S ( x)) ((x A) S ( x))


um axioma lgico, significando que os dois lados da implicao
tm sempre o mesmo valor de verdade, quando interpretadas em A:
ou ambas sero verdadeiras ou ambas, falsas. Outro axioma lgico
o seguinte
S (a ) (x A) S ( x) ,
supondo, agora, que a uma constante do conjunto numrico A.
Exemplo: (1.000.001 > 1.000.000) (x ) ( x > 1.000.000)
Observe, tambm, que

((x A) S ( x)) ((x A) S ( x))


Exemplo: nem todo nmero inteiro maior que zero,
((x ) x > 0) , o mesmo que existe um nmero inteiro que
menor ou igual a zero, ou seja, (x , x 0) .
Definio: uma frmula aberta S ( x) , precedida de um quantificador sobre a varivel x , chamada de sentena. Uma frmula sem
variveis tambm chamada de sentena.
Exemplos:
a) 1 + 2 = 3
b)
c) (y ) , y + 1 = 2
d) log (1) = 0
As frmulas abertas podem ter mais de uma varivel. Por exemplo,
sejam H = {Jorge, Cludio, Paulo}, M = {Suely, Carmen}. Seja S ( x, y )
a frmula aberta x irmo de y.
Ento S ( x, y ) uma frmula de duas variveis.

101

Definio: toda frmula aberta com duas (ou mais) variveis precedidas por quantificadores, um para cada varivel, chamada de
sentena.
Exemplos: a proposio

(x H ) ( y M ) S ( x, y )
pode-se ler Para cada x em H existe pelo menos um y em M tal que
x irmo de y, o que interpretada, em H M , do seguinte modo:
cada homem de H irmo de Suely ou de Carmem.
J a proposio

significa que existe uma mulher em M que irm de todos os homens de H. Esses dois exemplos mostram que mudando a ordem
dos quantificadores, obtm-se proposies diferentes. A negao da
proposio Para cada x em H existe pelo menos um y em M tal que
x irmo de y, ou, simbolicamente,

(x H ) ( y M ) S ( x, y ) ,
existe um x em H que no irmo de nenhum y em M, ou ainda,
existe x em H tal que para todo y em M, x no irmo de y, ou,
simbolicamente,

( x H ) ( y M ) ( S ( x, y )) .
Assim, se S uma frmula aberta de duas variveis ento

((x A) ( y B) S ( x, y )) ( x A) (y B) ( S ( x, y )) .
Isto significa que os dois lados da implicao tm sempre o mesmo
valor de verdade em A B .
Alm disso,
,

(( x A) ( y B) S ( x, y )) ( x A) (y B) ( S ( x, y ))
e

(( x A) ( y B) S ( x, y )) ( x A) ( y B) ( S ( x, y )) .

102

Exerccios
1) Seja A = {0,1, 2,3, 4,5} . Classifique as sentenas como verdadeiras ou falsas, justificando sua resposta.
a)
b) ( x A) x 2 1 = 3
c) ( x A) 3 x 6 = 14
d) (x A) x 5 < 1
2) Escreva a negao das sentenas do exerccio 1.
3) Sendo A = {0,1, 2,3, 4,5, 6} , coloque o quantificador adequado
para que a frmula se torne uma sentena verdadeira.
a) x + 4 = 8
b) x 2 5 x + 6 = 0
c) 5 x + 2 4
d) x 2 + 8 x = 0
4) Classifique as sentenas como verdadeiras ou falsas, justificando.
a) ( x ) ( y ) x + y = 3
b)
c) ( x ) ( y ) x 4 y > 0
d) ( x ) ( y ) x 2 y 3 < 0
5) Negue as proposies abaixo.
a) Todo estudante boa pessoa.
b) Todos os pssaros tm asas e voam.
c) Para cada nmero natural n , existe um nmero primo p
tal que p maior que n .
d) (x ) x + 3 > 8
e) ( x ) x 2 = 5

103

f) ((x ) x + 2 6) (( x ) x 2 5 = 4) (o smbolo denota o conectivo e)


g) (( x ) x 2 = 16) ((x ) x 1 = 8) (o smbolo denota o
conectivo ou)
h) (x ) (y ) cos ( x + y ) = cos ( x) cos ( y ) sen (x) sen (y )
i) ( x ) ( y ) log ( xy ) = log ( x) log ( y )
j) ( x ) ( y ) (3 x + 4 y = 7) ( x > y )
k)

4.6 Exerccios Propostos


1) Negue as afirmaes:
a) Esta rosa vermelha e tem espinhos.
b) Hoje segunda-feira.
c) x um nmero par ou x um nmero mpar.
d) x no um nmero primo e x um nmero par.
e) Durmo e sonho ou, ento, canto e dano.
2) Considerando as afirmaes: A: Max no bom motorista e
B: Clia boa cozinheira, escreva as proposies: ~A ou B; A e
~B; ~A e ~B; A B; B e ~A ; ~B ou A; ~A ou ~B.
3) Identifique hiptese e tese e d a contra-positiva das afirmaes abaixo:
a) Se a mulher dana tango, ento o marido torcedor do River Plate.
b) Se no est chovendo, ento est nevando.
c) Se ela no atende ao telefone, ento ela est passeando pela
praia ou est regando as flores no jardim.
d) Se voc bebe bastante gua e no come carne, ento voc
no tem olheiras.
e) Se Carlos for ao cinema, ento, se Paula no for, Quincas ir.

104
f) Se Csar ambicioso e Dbora no ciumenta, ento Csar
e Dbora formam um casal ideal.
g) Humberto come seu chapu se o Flamengo no ganhar um
jogo.
h) Se x e y so nmeros reais, e x.y = 0, ento x = 0 ou y = 0.
i) Dois nmeros naturais a e b so relativamente primos somente se o mximo divisor comum deles 1.
j) Um tringulo eqiltero se todos os seus lados so iguais.
k) Duas retas distintas no espao so paralelas se elas no so
concorrentes e no so reversas.
l) Um nmero racional se pode ser expresso como razo de
inteiros a/b, com b diferente de zero.
m) Um inteiro a divisvel por um inteiro b somente se a mltiplo de b.
n) Um nmero mltiplo de 5 se seu algarismo das unidades
zero ou 5.
o) Se uma girafa tem dor de garganta, ento ela no come acerola.
p) Um tringulo retngulo se contm um ngulo de 90.
q) Irei praia desde que no chova.
r) Se uma ave no um pelicano, ento ela no come peixe.
s) Se x um inteiro mpar, ento x da forma 2k+1 para algum
k inteiro.
t) Se x par e primo, ento x igual a 2.
u) Se x da forma 3k+1 ou 3k+2, para algum inteiro k, ento x
no mltiplo de 3.
v) x da forma 2k ou 2k+1, para algum inteiro k, se x inteiro.
w) Se no venta e no chove, Edmundo joga na Ressacada.
x) Se voc jardineiro, ento sabe plantar e podar.
y) Se a lua est cheia, os vampiros saem de casa noite.

105

z) Se voc tem mais de 90 anos, a Academia do Chope lhe d


lies de aerbica grtis.
4) D a recproca das afirmaes do exerccio 3.
5) Negue as afirmaes do exerccio 3.
6) Negue as afirmaes abaixo:
a) Se as diagonais de um paralelogramo so perpendiculares
uma outra, ou se elas dividem seus ngulos em duas partes iguais, ento o paralelogramo um losango.
b) Se um de dois nmeros um inteiro mltiplo de 3 e, se a
soma dos dois nmeros um inteiro mltiplo de 3, ento o
outro nmero mltiplo de 3.
c) Se um paralelogramo no um losango, ento suas diagonais no so perpendiculares uma outra, ou elas dividem
seus ngulos em duas partes iguais.
d) Se um nmero a igual a zero ou um nmero b igual a
zero, ento o nmero ab igual a zero.
e) Se o nmero ab diferente de zero, ento, o nmero a diferente de zero e o nmero b diferente de zero.
f) Se o nmero a maior que zero e o nmero b maior que
zero, ento o nmero ab maior que zero.
7) Negue as bi-condicionais:
a) Um nmero mltiplo de 5 se, e somente se, divisvel
por 5.
b) Uma figura plana um paralelogramo se, e somente se,
um quadrado.
c) A soma dos ngulos internos de um tringulo 180 se, e
somente se, o tringulo issceles.
d) O nmero a.b igual a zero se, e somente se, o nmero a
igual a zero ou o nmero b igual a zero.

106
8) Em cada um dos raciocnios abaixo, diga se a terceira afirmao conseqncia das duas primeiras, usando a regra Modus
Ponens.
a) Se voc esqueceu seu lpis, pode pegar um dos meus. Voc
esqueceu seu lpis. Logo, pode pegar um dos meus.
b) Se voc v manchas na sua frente, ento voc est olhando
um tigre. Voc est olhando um tigre. Logo, voc v manchas na sua frente.
c) Se voc escova seu dente com Xampu Maravilha, ento voc
no entendeu o comercial. Voc entendeu o comercial. Logo,
voc no escova seus dentes com Xampu Maravilha.
d) Pedras que rolam no criam limo. Se as pedras no criam
limo, ento elas so lisas. Logo, se as pedras rolam, elas so
lisas.
e) Se voc l Veja, ento voc gosta de gatos. Se voc cria passarinhos, ento no gosta de gatos. Logo, se voc cria passarinhos, no l Veja.
f) Se voc andar debaixo de um coqueiro, provavelmente ter
sua cabea rachada. Quem vai a Fortaleza anda debaixo de
coqueiros. Logo, se voc for a Fortaleza ter sua cabea rachada, provavelmente.
g) Picadinho melhor do que nada. Nada melhor do que
feijoada. Logo, picadinho melhor do que feijoada.
h) Se Pedro ganha no bingo, ento Antnio bebe cerveja. Se
Antnio bebe cerveja, sua namorada briga. Se a namorada
briga, Antnio fica triste. Antnio no est triste. Logo, Pedro no ganhou no bingo.
i) No posso ajud-lo se eu no souber o que est errado e ainda no sei o que est errado. Logo, no posso ajud-lo.
j) Se um cachorro bravo, ento ele late muito. Se um cachorro late muito, ele incomoda os vizinhos. Euclides um cachorro bravo. Logo, ele incomoda os vizinhos.
9) D dois exemplos de proposies verdadeiras do tipo se .. ento (em interpretaes usuais), tais que a recproca de uma
seja verdadeira e a recproca da outra seja falsa.

Se A B uma
proposio verdadeira e A
uma proposio tambm
verdadeira, ento B uma
proposio verdadeira.

107

10) Considere os quadrilteros definidos abaixo:


Trapezide - quadriltero sem lados paralelos.
Trapzio - quadriltero tendo exatamente dois lados
paralelos.
Trapzio issceles - trapzio com dois lados opostos congruentes (e no paralelos).
Trapzio retngulo - trapzio com ngulo reto.
Paralelogramo - quadriltero com dois pares de lados opostos
paralelos.
Retngulo - paralelogramo com ngulo reto.
Losango - paralelogramo com todos os lados congruentes.
Quadrado - retngulo e losango.
Represente num diagrama de Euler como se relacionam os
quadrilteros definidos acima.
11) Resolva a equao:

x 2 + 1 = x 1 . Por que zero no soluo?

12) Considere a afirmao: Se um tringulo eqiltero, ento


todos os seus lados so iguais. Discuta a veracidade da recproca. O que voc pode concluir sobre proposies que so
definies?
Monty Python em busca
do clice sagrado (Monty
Python and the Holy Grail)

13) O raciocnio abaixo foi extrado de um filme do grupo Monty


Python, no qual h um julgamento em praa pblica para se
decidir se uma mulher A feiticeira ou no.
Feiticeiras so queimadas, assim como madeira. Basta ver ento se A de madeira. Para isso no adianta tentar construir
uma ponte com A porque existem pontes de pedra. melhor
ver se A flutua, como a madeira. Como patos tambm flutuam,
basta ver se A pesa o mesmo que um pato. Se isso acontecer A
feiticeira. Descubra todos os erros nesta cadeia de argumentos, se que existe algum.
14) Negue as sentenas seguintes (com quantificadores):
a) Todo homem careca barrigudo.

108

b) Existe uma rvore com 12 metros de altura.


c) Todos os nmeros quadrados so no primos.
d) Existem mamferos que so felinos.
e) Todo homem incompetente fracassa em ser feliz.
f) Alguns gnus so ferozes.
g) Para todo nmero inteiro x, se x > 2, ento x 2 > 0.
h) Se A, B e C so pontos colineares e B est entre A e C, ento
toda reta que passa por B corta o segmento AC.
i) Todo homem chora de tristeza ou de raiva.
j) Todo brasileiro rico, feliz e vive no Rio.
k) Toda resposta est certa e errada.
15) Verifique a veracidade das afirmaes abaixo e faa a negao
de cada uma delas:
a)

b)

c)

d) (m ) ( n ) m > n .
e) ( n ) (m ) m m > n .
16) Decida que concluses so vlidas:
a) Todos os girassis so amarelos e alguns pssaros so amarelos. Logo, nenhum pssaro um girassol.
b) Nenhum advogado uma pessoa entusiasmada. Voc
uma pessoa entusiasmada. Logo, voc no advogado.
c) Dicionrios so teis e livros teis so valiosos. Logo, dicionrios so valiosos.
d) Tudo o que ele fala tolice. Toda tolice desprezvel. Logo,
tudo que ele fala desprezvel.
e) Ningum conquistou o mundo. Logo, algum no conquistou o mundo.

109

f) Como todos os briques so braques, todos os braques so


trecos e existem minerais que so briques, podemos concluir que existem minerais que so trecos.
g) Nenhum astro que brilha com luz prpria planeta. Todas
as estrelas so astros dotados de luz prpria. Logo, nenhuma estrela planeta.
h) Alguns chimpanzs so agressivos. Alguns animais agressivos atacam o homem. Logo, alguns chimpanzs atacam o
homem.
17) Deduza, se possvel, alguma concluso das sentenas abaixo:
a) Algumas lies so difceis. O que difcil necessita ateno.
b) Todos os britnicos so corajosos. Nenhum marinheiro covarde.
c) Alguns sonhos so terrveis. Nenhum carneiro terrvel.
d) Nenhuma pessoa autoritria popular. Simo autoritrio.
e) Nenhum beb estudioso. Todos os violonistas so estudiosos.
f) Todos os filsofos so lgicos. Todas as pessoas lgicas so
distradas.
g) Todo nmero natural inteiro. O nmero 567 um nmero
natural.
h) Alguns santistas so surfistas. Alguns senadores so santistas.
i) A pressa inimiga da perfeio. Todas as tartarugas so
lentas.
j) Todos os homens so mortais. O presidente homem.
k) Todas as borboletas podem voar. Alguns porcos no podem
voar.
l) Bebs no so lgicos. Nenhuma pessoa descuidada pode
treinar um crocodilo. Pessoas que no so lgicas so descuidadas.

110

m) Toda gildebnia mancspia. Nenhuma mancspia gorda.


n) Alguns violinistas tm longos dedos. Todas as pessoas de
longos dedos tm enxaqueca.
o) Todo xintap calrio. Todo calrio pedicular.
p) Todo gato de trs cores fmea. Manhoso um gato que
tem trs cores.
q) Todos os mamferos so animais de sangue quente. Nenhum lagarto animal de sangue quente.
r) Alguns insetos tm oito patas. Todos os insetos de oito patas so venenosos.
s) Alguns gatos so brancos. Alguns gatos so ferozes.
18) O romance Dom Quixote de la Mancha, de Miguel de Cervantes, fala-nos de uma ilha onde vigora uma lei curiosa. Um
guarda pergunta a cada visitante: Por que voc vem aqui?
Se o visitante responder a verdade, est salvo. Se no, ser enforcado. Um dia, um visitante respondeu: Vim aqui para ser
enforcado! Conseguiremos saber o que aconteceu a ele?

Referncias
[1] BEZERRA, Licio H. et al. Introduo Matemtica. Florianpolis:
Ed. UFSC, 1995.
[2] MORTARI, Cezar A. Introduo Lgica. So Paulo: UNESP, 2001.
[3] ABAR, Celina A.A.P. Noes de Lgica Matemtica. Disponvel em:
<www.pucsp.br/~logica/>. Acesso em: 01 JUN 2009.

111

Sugestes de leitura
[1] DEWDNEY, A. K. 20 000 lguas matemticas. Rio de Janeiro: Jorge
Zahar, 2000.
[2] GUZMAN, Miguel de. Aventuras matemticas. Lisboa: Gradiva,
1991.
[3] KAPLAN, Robert. O nada que existe. Rio de Janeiro: Rocco, 2001.
[4] MORRIS, Richard. Uma breve histria do infinito. Rio de Janeiro:
Jorge Zahar, 1998.
[5] OSSERMAN, Robert. A magia dos nmeros no universo. So Paulo:
Mercuryo, 1997.
[6] SMULLYAN, Raymond. O enigma de Sherazade e outros incrveis
problemas: das Mil e uma noites lgica moderna. Rio de Janeiro: Jorge Zahar, 1998.

112

Bibliografia comentada
GARDNER, Martin. Divertimentos Matemticos. 4. ed. So Paulo:
Ibrasa, 1998.
Martin Gardner assinava a coluna Mathematical Games, da revista Scientific
American, nas dcadas de 60 e 70. Tem vrios livros editados em ingls.
Em portugus, a editora portuguesa Gradiva publicou diversos ttulos. Esse
livro da editora brasileira IBRASA para quem gosta especialmente de jogos
e quer calcular ou conhecer, pelo menos, as possibilidades matemticas de
desenvolvimento de uma partida. O livro trata de variaes do jogo da
velha, apresenta o jogo chins do NIM e muitas outras curiosidades, como
variaes de quadrados mgicos.

GARDNER, Martin. O festival Mgico da Matemtica. Lisboa: Gradiva, 1994.


Polimins, nmeros perfeitos, nmeros amigveis, nmeros sociveis,
aritmtica digital, rvores. Voc encontra todos estes assuntos no livro.

MALBA TAHAN. O homem que calculava. 56. ed. Rio de Janeiro: Record, 2002.
O nosso querido matemtico brasileiro escreve um romance sobre um
viajante por terras estranhas, chamado de o calculista. So contos
interligados, no estilo dos Contos de mil e uma noites, cheios de califas,
mulheres com burcas, comerciantes, sempre com algum problema
matemtico para ser resolvido. E o nosso heri resolve todos, com solues
cheias de humor, como os contos de Nasruddin. No deixe de t-lo em sua
biblioteca particular.

Você também pode gostar